BEC198 (Finals)

You might also like

Download as docx, pdf, or txt
Download as docx, pdf, or txt
You are on page 1of 180

Basic Engineering Correlation (Algebra Reviewer)

1. Three transformers are directly proportional to the KVA cost P30,000. The cost of each transformer is directly
proportional to the KVA rating and each has a constant of proportionally of 0.9, 0.8 and 0.6, respectively. Find the
cost of the KVA transformer.
a. P7,500
b. P13,500
c. P15,500
d. P9,000

2. What is the sum of the following sequence of terms 18, 25, 32, 39, . . . ,67?
a. 280
b. 380
c. 320
d. 340

3. A train, an hour after starting, meets with an accident which detains it an hour, after which it proceeds at 3/5 of
its former rate and arrives three hour after the time; but had the accident happened 50 miles farther on yhe line, it
would have arrived one and one-half hour sooner. Find the length of the journey.
a. 850/9 miles
b. 800/9 miles
c. 920/9 miles
d. 910/9 miles

4. Ten less than four times a certain number is 14. Determine the number.
a. 5
b. 7
c. 4
d. 6

5. The roots of a quadratic equation are 1/3 and 1/4. What is the equation?
a. 12x2 + 7x + 1=0
b. 12x2 - 7x - 1=0
c. 12x2 - 7x + 1=0
d. 12x2 + 7x - 1=0

6. The geometric mean of 4 and 64:


a. 30
b. 34
c. 24
d. 16

7. A certain company manufactures two products, X and Y, and each of these products must be processed on two
different machines. Product X requires 1 minute of work time per unit on machine 1 and 4 minutes of work time on
machine 2. Product Y requires two minutes of work time per unit on machine 1 and 3 minutes of work time per
unit on machine 2. Each day, 100 minutes are available on machine 1 and 200 minutes are available on machine 2.
To satisfy certain customers, the company must produce at least 6 units per day of product X and at least 12 units
of product Y. If the profit of each unit of product X is P50 and the profit of each unit of product Y is P60, how many
of each product should be produced in order to maximize the company's profit?
a. X = 20 units, Y = 40 units
b. X = 30 units, Y = 40 units
c. X = 20 units, Y = 50 units
d. X = 40 units, Y = 60 units
8. If 4y3 + 18y2 + 8y - 4 is divided by 2y + 3, the remainder is:
a. 10
b. 12
c. 11
d. 9

9. The square of a number increased by 16 is the same as 10 times the number. Find the number.
a. 8, 2
b. 6, 2
c. 4, 2
d. 2, 2

10. The seating section in a coliseum has 30 seats in the first row, 32 in the second row, 34 seats in the third row
and so on, until the tenth row is reached, after which there are ten rows each containing 50 seats. Find the total
number of seats in the section.
a. 1290
b. 1080
c. 890
d. 980

11. If the roots of an equation is zero, then they are classified as


a. hypergolic solutions
b. trivial solutions
c. conditional solutions
d. extraneous solutions

12. An airplane went 360 miles in 2 hours with the wind and, flying back the same route, it took 3 3/5 hours against
the wind. What was its speed in still air?
a. 120 mph
b. 150 mph
c. 140 mph
d. 130 mph

13. Find the fourth proportion to 3, 5 and 21.


a. 27
b. 65
c. 56
d. 35

14. Two jet planes travelling towards each other take off at the same time from two airports located 3000 miles
apart. If they passed each other after two hours, determine the speed of each plane if one plane is flying at a speed
100 mph faster than the other.
a. 700 and 800 mph
b. 600 and 700 mph
c. 700 and 900 mph
d. 800 and 500 mph

15. Round off 0.003086 to three significant figures.


a. 0.0031
b. 0.00308
c. 0.003
d. 0.00309
16. It is sequence of numbers that successive terms differ by a constant.
a. geometric progression
b. arithmetic progression
c. harmonic progression
d. finite progression

17. At 2:00 pm, an airplane takes off at 340 mph on an aircraft carrier. The aircraft carrier moves due south at 25
kph in the same direction as the plane. At 4:05 pm, the communication between the plane and aircraft carrier was
lost. Determine the communication range in miles between the plane and the carrier.
a. 785 miles
b. 557 miles
c. 412 miles
d. 656 miles

18. A manufacturing firm maintains one product assembly line to produce signal generators. Weekly demand for
the generators is 25 units. The line operates for 7 hours per day, 5 days per week. What is the maximum
production time per unit in hours required for the line to meet the demand?
a. 3 hours
b. 1 hour
c. 2.25 hours
d. 0.75 hour

19. Ana is 5 years older than Beth. In5 years, the product of their age is 1.5 times the product of their product ages.
How old is Beth now?
a. 20
b. 25
c. 18
d. 27

20. A chemist of a distillery experimented on two alcohol solutions of different strengths, 30% alcohol and 60%
alcohol, respectively. How many cubic meters of each strength must be used in order to produce a mixture of 50
cubic meters that contain 40% alcohol?
a. 20, 30 m3
b. 33 1/3, 16 2/3 m3
c. 21 1/3, 28 2/3 m3
d. 10, 40 m3

21. Subtracting 2.6 x 103 from8.26 x 104 is:


a. 8.0 x 104
b. 10.86 x 104
c. 8.0 x 103
d. 10.86 x 103

22. The time requires by an evaluator to lift a weight varies directly with the weight and the distance through
which it is to be lifted and inversely as the power of the motor. If it takes 30 seconds for 10 hp motor to lift 100 lbs
through 50 feet, what size of motor is required to lift 800 lbs in 40 seconds through a distance of 40 feet?
a. 56 hp
b. 50 hp
c. 58 hp
d. 48 hp
23. Find the 30th term of the arithmetic progression 4, 7, 10, . . .
a. 94
b. 941
c. 81
d. 104

24. Convergent series is a sequence of decreasing numbers or when the succeeding term is _______ than the
preceding term.
a. equal
b. slightly more
c. greater
d. lesser

25. In the equation x2 + x = 0, one root is x equal to:


a. 1
b. ¼
c. 5
d. none of these.

26. How many liters of water must be added to 35 liters of 89% hydrochloric acid solution to reduce its strength to
75%?
a. 4.83 liters
b. 6.53 liters
c. 7.33 liters
d. 5.34 liters

27. Round off 34.2814 to four significant figures.


a. 34.8214
b. 34
c. 34.28
d. 34.281

28. Solve algebraiclly:


11y2 - 3x2 = 41
4x2 + 7y2 = 32.
a. (-2, 2) and (2, -2)
b. (± 1, ± 2)
c. (± 1, ± 4)
d. (2, 3)and ( -2, -3)

29. Determine the sum of the progression if there are 7 arithmetic means between 3 and 35.
a. 98
b. 304
c. 214
d. 171

30. Crew No. 1 can finish installation of an antenna tower in 200 man-hour while Crew No. 2 can finish the same
job in 300 man-hour. How long will it take both crews to finish the same job, working together?
a. 120 man-hour
b. 140 man-hour
c. 100 man-hour
d. 160 man-hour
31. In how many minutes after 3:00 P.M will the minute hand of a clock coincide with the hour hand?
a. 15.455
b. 17.273
c. 16.364
d. 18.182

32. In a class of 40 students, 27 students like Calculus and 25 like Geometry. How many students liked both
Calculus and Geometry?
a. 12
b. 13
c. 11
d. 10

33. The electric power which a transmission line can transmit is proportional to the product of its design voltage
and current capacity, and inversely to the transmission distance. A 115 - kilovolt line rated at 100 amperes can
transmit 150 megawatts over 150 km. How much power, in megawatts can a 230 kilovolt line rated at 150 amperes
transmit over 100 km?
a. 595
b. 675
c. 485
d. 785

34. The electrical resistance of a wire varies as its length and inversely as the square of its diameter. If a 100 m long
and 1.25 mm in diameter has a resistance of 30 ohms, find the length of the wire of the same material whose
resistance and diameter are 25 ohms and 0.74 mm respectively.
a. 25 m
b. 35 m
c. 30 m
d. 40 m

35. What time after 3 o'clock will the hands of the clock be together for the first time?
a. 3:02.30
b. 3:17.37
c. 3:16.36
d. 3:14.32

36. A pump can pump out water from a tank in 11 hours. Another pump can pump out water from the same tank
in 20 hours. How long will it take both pumps to pump out water in the tank?
a. 6 hours
b. 6 1/2 hours
c. 7 1/2 hours
d. 7 hours

37. If the sum is 220 and the first term is 10, find the common difference if the last term is 30.
a. 3
b. 4
c. 5
d. 2

38. Equal volumes of two different liquids evaporated at different but constant rates. If the first is totally
evaporated in 6 weeks and the second in 5 weeks, when will the second be one-half the volume of the first?
a. 3.5 weeks
b. 3 weeks
c. 4 weeks
d. 4 2/7 weeks

39. MCMXCIV is a Roman numeral equivalent to:


a. 1994
b. 2174
c. 3974
d. 2974

40. Find the 100th term of the sequence 1.01, 1.00, 0.99, . .
a. 0.01
b. 0.02
c. 0.03
d. 0.04

41. At what time after 12:00 noon will the hour hand and minute hand of the clock first form an angle of 120 o?
a. 12:21.818
b. 12:22.818
c. 12:18.818
d. 12:24.818

42. Solve the simultaneous equations:


3x - y = 6
9x - y = 12.
a. ( -1, 3 )
b. ( 1, -3 )
c. ( 1, 3 )
d. ( -1, -3 )

43. A merchant has three items on sale: namely, a radio for P50, a clock fo P30, and a flashlight for P1. At the end
of the day, she has sold a total of 100 of the three items and has taken exacly P1000 on the total sales. How many
radios did he sale?
a. 4
b. 80
c. 20
d. 16

44. What is the sum of the first 10 terms of the geometric progression 2, 4, 8, 16, . . . ?
a. 1696
b. 2046
c. 1024
d. 1846

45. In a commercial survey involving 1000 persons on brand preferences, 120 were found to prefer brand x only,
200 persons prefer brand y only, 150 persons prefer brand z only, 370 prefer either brand x or y but not z, 450
prefer brand y or z but not x, and 370 prefer either brand z or x but not y, and none prefer all the three brands at a
time. How many persons have no brand preference with any of the three brands?
a. 200
b. 100
c. 280
d. 70

46. Which number has four significant figures?


a. 1.414
b. 0.0014
c. 0.141
d. 0.01414

47. A club of 40 executives, 33 likes to smoke Marlboro and 20 likes to smoke Philip Morris. How many like both?
a. 12
b. 13
c. 14
d. 11

48. The arithmetic mean of 80 numbers is 55. If two numbers namely 250 and 850 are removed, what is the
arithmetic mean of the remaining numbers?
a. 41.25
b. 42.31
c. 44.25
d. 40.21

49. There are 9 arithmetic means between 11 and 51. The sum of the progreesion is:
a. 374
b. 341
c. 320
d. 337

50. If a two digit number has X for its unit digit and Y for its tenth digit, represent the number.
a. 10Y + X
b. X + Y
c. XY
d. 10Y + Y

51. In the series 1, 1, 1/2, 1/6, 1/24, . . . , determine the 6 th term.


a. 1/60
b. 1/120
c. 1/150
d. 1/90

52. Round off 149.691 to the nearest integer.


a. 149
b. 149.7
c. 149.69
d. 150

53. The sum of two numbers is 21, and one number twice the other. Find the numbers.
a. 9 & 12
b. 7 & 14
c. 8 & 13
d. 65 & 70

54. The probability for the ECE board examinees from a certain school to pass the Mathematics subject is 3/7 and
that for the Communication subject is 5/7. If none of the examinees failed in both subjects, how many examinees
from the school took the examination?
a. 30
b. 27
c. 29
d. 28

55. Solve for x that satisfies the equation 6x2 - 7x - 5 = 0.


a. 3/5 or ¾
b. 3/2 or 3/8
c. 5/3 or -1/2
d. 7/5 or -7/15

56. Three transformers are rated 5 KVA, 10 KVA and 25 KVA, respectively. The total cost of the three transformers
is P15, 000.00. If the cost of each transformer is proportional to its KVA rating multiplied by the factor 1, 0.8 and
0.6 respectively, find the cost of the 10 KVA transformer.
a. P4,286
b. P4,075
c. P4,101
d. P4,393

57. Solve the simultaneous equations:


2x2 - 3y2 = 6
3x2 + 2y2 = 35.
a. x-3 or 3; y2 or -1
b. x3 or -3; y2 or -2
c. x3 or -3; y-2 or 1
d. x3 or -3; y-2 or 3

58. The sum of the progression 5, 8, 11, 14, . . . Is 1025. How many terms are there?
a. 25
b. 24
c. 28
d. 29

59. If x varies directly as y and inversely as z, and x = 14 when y = 7 and z = 2, find the value of x when y = 16 and z =
4.
a. 4
b. 8
c. 16
d. 14

60. The arithmetic means of 6 numbers is 17. If two numbers are added to the progression, the new set of the
numbers will have an arithmetic mean of 19. What are the two numbers if their difference is 4?
a. 18, 22
b. 23, 27
c. 10, 14
d. 31, 35

61. The sum of Kim's and Kevin's ages is 18. In 3 years, Kim will be twice as old as Kevin. What are their ages now?
a. 5, 13
b. 7, 11
c. 6, 12
d. 4, 14

62. The intensity of sound varies directly as the strength of the source and inversely as the square of the distance
from the source. Write the equation to the describe relation.
a. I = 1/d2 + k
b. I=k/d2
c. I = kd2
d. I = d2/k

63. Determine the sum of the infinite series 1/3 + 1/9 + 1/27 +. . .
a. 1
b. ¾
c. ½
d. 2/3

64. For a particular experiment, you need 5 liters of 10% solution. You find 7% and 12% solutions on the shelf. How
much of the 7% solution you mix with the appropriate amount of the 12% solution to get 5 liters of 10% solution?
a. 2.5
b. 2
c. 1.5
d. 3

65. Find the sum of the roots of 5x2 - 10x + 2 = 0


a. -2
b. ½
c. -1/2
d. 2

66. Maria is 36 years old. Maria was twice as old as Anna was when Maria was as old as Anna is now. Jow old is
Anna now?
a. 26
b. 31
c. 29
d. 24

67. Find the ratio of an infinite geometric progression if the sum is 2 and the first term is 1/2.
a. 2/3
b. 1/6
c. ¾
d. ¼

68. A tank is fitted with two pipes. The first pipe can fill the tank in 10 hours. But after it has been open for 3 hours,
the second pipe is opened and the tank is filled up in 4 hours more. How long would it take the second pipe alone
to fill tha tank?
a. 12.67 hr
b. 10.55 hr
c. 14.89 hr
d. 13.33 hr

69. How many kg of cream containing 25% butter fat should be added to 50 kg of milk containing one percent
butter fat to produce milk containing 2% butter fat?
a. 4.17
b. 2.174
c. 5.221
d. 3.318
70. The electrical resistance offered by an electric wire varies directly as the length and inversely as the square of
the diameter of the wire. Compare the electrical resistance offered by two pieces of wire of the same material, one
being 100 m long and 8 mm in diameter, and the other 50 m long and 3 mm in diameter.
a. R1 = 0.28 R2
b. R1 = 0.84 R2
c. R1 = 0.57 R2
d. R1 = 0.95 R2

71. A stack of bricks has 61 bricks in the bottom layer, 58 bricks in the second layer, 55 bricks in the third layer, and
so on until there are 10 bricks in the last layer. How many bricks are there all together?
a. 458
b. 639
c. 724
d. 538

72. A 100 g of water are mixed with 150 g of alcohol (p = 790 kg/cu.m.). What is the specific volume of the resulting
mixtures? Assuming that the two fluids mix completely.
a. 0.63 cu cm/g
b. 0.88 cu. cm/g
c. 0.82 cu cm/g
d. 1.20 cu cm/g

73. One number is 5 less than another. If the sum is 135, what are the numbers?
a. 65, 70
b. 60, 65
c. 75, 80
d. 70, 75

74. The denominator of a certain fraction is three more than twice the numerator. If 7 is added to both terms of
the fraction, the resulting fraction is 3/5. Find the original fraction.
a. 8/5
b. 13/5
c. 5/13
d. 3/5

75. An inexperienced statistical clerk submitted the following statistics to his manager on the average rate of
production of transistorized radios in an assenbly line: "1.5 workers produced 3 radios in 2 hour." How many
workers are employed in the assembly line working 40 hours per week if weekly production is 480 radios?
a. 12
b. 10
c. 13
d. 14

76. Find the mean proportion of 4 and 36.


a. 12
b. 8
c. 16
d. 9

77. An automobile is travelling at a velocity of 10 mph. If the automobile mileage meter already reads 20 miles,
find the mileage meter reading after 3 hours.
a. 60 miles
b. 30 miles
c. 50 miles
d. 40 miles

78. Find the sum of 1, -1/5, 1/25, . . .


a. 6/7
b. 7/8
c. 5/6
d. 8/9

79. A man is 41 years old and his son is 9. In how many years will the father be three times as old as his son?
a. 7
b. 8
c. 6
d. 5

80. A tank is fitted with an intake pipe that will fill it in 4 hours, and an outlet pipe that will empty it in 9 hours. If
both pipes are left open, how long will it take to fill the empty tank?
a. 7.2 hr
b. 6.8 hr
c. 6.2 hr
d. 7.4 hr

81. Find the 1987th digit in the decimal equivalent to 1785/9999 starting from the decimal point.
a. 1
b. 5
c. 7
d. 8

82. A mechanical engineer who was awarded a P450,000.00 contract to install the machineries of an oil mill failed
to finish the work on time. As provided for in the contract, he has to pay a daily penalty equivalent to one-fourth of
one percent of the contract price for the first ten days of the delay, one-half percent per day for the next ten days
and one percent per day for every day thereafter. If the total penalty paid was P60,750.00, how many days was the
completion of the contract delayed?
a. 30 days
b. 26 days
c. 24 days
d. 28 days

83. A man started driving his car at a certain time froma certain place. On arrival at his destination at the precise
appointed time, he said, "If I had averaged 6 miles per hour faster, I would have been 5 minutes early. But if I had
averaged 5 mph slower, I would have been 6 minutes late." Find how far he had driven.
a. 20 miles
b. 10 miles
c. 25 miles
d. 15 miles

84. Pedro started running at a speed of 10kph. Five minutes later, Mario started running in the same direction and
catches up with Pedro in 20 minutes. What is the speed of Mario?
a. 12.5 kph
b. 17.5 kph
c. 20.5 kph
d. 15.0 kph
85. The equation whose roots are the reciprocal of the solutions of 2x2 - 3x - 5 = 0.
a. 3x2 - 5x - 2=0
b. 5x2 - 2x - 3=0
c. 5x2 + 3x - 2=0
d. 2x2 + 5x - 3=0

86. In certain Board Examination, 119 examinees too the Shop Machinery subjected, 104 examinees took thye
Power Plant Machinery subject and 115 examinees took the Industrial Plant Machinery subject. Seventy-eight (78)
conditioned examinees took only Shop Machinery and Power Machinery subjects. Seventy-one (71) conditioned
examinees took only the POwer Plant Machinery and Industrial Plant Machinery subjects. Eighty-five (85)
conditioned examinees took only Industrial Plant Machinery and Shop Machinery subjects. Fifty-four took all the
three subjects. How many examinees took the Certified Plant Mechanic board examination?
a. 153
b. 165
c. 158
d. 176

87. If a train passes as many telegraph poles in one minute as it goes miles per hour, how far apart are the poles?
a. 78 ft.
b. 98 ft.
c. 68 ft.
d. 88 ft.

88. A man 38 years old has a son of ten years old. In how many years will the father be three times as old as his
son?
a. 2
b. 3
c. 4
d. 5

89. In Algebra, the operation of root extraction is called as _____.


a. revolution
b. resolution
c. involution
d. evolution

90. Pedro can paint a fence 50% faster than Juan and 20% faster than Pilar and together they can paint a given
fence in 4 hours. How long will it take Pedro to paint the same fence if he had to work alone?
a. 15
b. 13
c. 10
d. 11

91. There are 9 arithmetic means between 11 and 51. The sum of the progression is:
a. 374
b. 341
c. 320
d. 337

92. The number 1.123123123. . . Is


a. surd
b. transcendental
c. rational
d. irrational

93. Which of the following numbers should be changed to make all the numbers from an arithmetic progression
when properly arranged?
a. 27/14
b. 45/28
c. 20/14
d. 3/28

94. How many significant digits do 10.097 have?


a. 4
b. 5
c. 2
d. 3

95. Find the sum of the infinite geometric progression 6, -2, 2/3, . . .
a. 9/2
b. 7/2
c. 3/2
d. 11/2

96. The time required for two examinees to solve the same problem differ by two minutes. Together they can solve
32 problems in one hour. How long will it take for the slower problem solver to solve the problem?
a. 3 minutes
b. 5 minutes
c. 2 minutes
d. 4 minutes

97. An equipment installation job in the completion stage can be completed in 50 days of 8 hours day work, with
50 men working. With the contract expiring in 40 days, the mechanical engineer contractor decided to add 15 men
on the job, overtime not being permitted. If the liquidated damages is P5,000 per day of delay, and they are paid
P150 per day, will the engineer be able to complete the job on time? Would he save money with the addition of
workers?
a. No, P20,500 losses
b. Yes, P44,750 savings
c. Yes, P24,500 savings
d. No, P15,750 losses

98. An airplane flying with the wind, took 2 hours to travel 1000 km and 2.5 hours in flying back. What was the
wind velocity in kph?
a. 40
b. 70
c. 60
d. 50

99. If a = b, then b = a. This illustrates which axiom in Algebra?


a. Transitive Axiom
b. Reflexive Axiom
c. Symmetric Axiom
d. Replacement Axiom

100. The ten's digit of a certain two digit number exceeds the unit's digit by four and is one less than twice the
unit's digit. Find the number.
a. 59
b. 95
c. 65
d. 85

101. One pipe can fill a tank in 6 hours and another pipe can fill the same in tank in 3 hours. A drain pipe can empty
the tank in 24 hours. With all three pipes open, how lomg will it take to fill in the tank?
a. 2.18 hrs
b. 2.23 hrs
c. 2.81 hrs
d. 2.32 hrs

102. An equipment installation job in the completion stage can be completed in 40 days of 8 hours day work with
40 men working. With the contract expiring in 30 days, the mechanical engineer contractor decided to add 10 men
on the job, overtime not being permitted. If the liquidated damages is P2,000 per day of delay, and the men are
paid P80 per day, will the engineer be able to complete the job on time?
a. No, there would be no savings
b. No, P16,000 would be lost
c. Yes, there would just be break even
d. Yes, P16,000 would be saved

103. It takes Butch twice as it takes Dan to do a certain piece of work. Working together they can do the work in 6
days. How long would it take Dan to do it alone?
a. 12 days
b. 9 days
c. 10 days
d. 11 days

104. Robert is 15 years older than his brother Stan. However, "y" years ago, Robert was twice as old as Stan. If Stan
is now "b" years old b.y, find the value of (b-y).
a. 18
b. 17
c. 15
d. 16

105. Mike, Loui and Joy can mow the lawn in 4, 6 and 7 hours, respectively. What fraction of the yard can they
mow in 1 hour if they work together?
a. 47/84 hr
b. 84/47 hr
c. 34/60 hr
d. 45/84 hr

106. The volume of hemisphere varies directly as the cube of its radius. The volume of a sphere with 2.54 cm
radius is 20.75 cm3. What is the volume of a sphere with 3.25 cm radius of the same kind?
a. 4056 cm3
b. 45.98 cm3
c. 43.47 cm3
d. 39.20 cm3

107. Add the following and express in meters: 3 m + 2 cm + 70 mm.


a. 3.14 m
b. 2.90 m
c. 3.12 m
d. 3.09 m

108. From the time 6:15 PM to the time 7:45 PM of the same day, the minute hand of a standard clock describe an
arc of:
a. 90o
b. 60o
c. 540o
d. 180o

109. A clock has dial face 304.80 mm in radius. The minute hand is 228.60 mm long while the hour hand is 152.40
mm long. The plane of rotation of the minute hand is 50.80 mm above the plane of rotation of the hour hand. Find
the distance between the tips of the hands of the clock at 5:40 AM.
a. 228 mm
b. 239 mm
c. 243 mm
d. 233 mm

110. A certain manufactured part can be defective because it has one or more out of the three possible defects:
insufficient tensile strength, a burr, or a diameter outside of tolerance limit. In a lot of 500 pieces: 19 have a tensile
strength defects, 17 have a burr, 11 have an unacceptable diameter, 12 have tensile strength and burr defects, 7
have tensile strength and diameter defects, 5 have burr and diameter defects and 2 have all three defects.
Determine:
How many of the pieces have no defects?
How many pieces have only burr defects?
How many pieces have exactly 2 defects?
a. 475, 2, 18
b. 490, 4, 10
c. 465, 3, 7
d. 480, 4, 6

111. Mary is 24 years old. Mary is twice as old as Ana waswhen Mary was as old as Ana is now. How old is Ana?
a. 18
b. 16
c. 20
d. 19

112. The electrical resistance of wire made of a certain material varies as its length and inversely as the square of
the diameter. If the wire 200 meters long and 1.25 mm in diameter has a resistance of 60 ohms, find the length of
the wire of the same material, whose resistance and diameter are 5 ohms and 0.65 mm, respectively.
a. 3.96 m
b. 4.51 m
c. 4.28 m
d. 5.72 m

113. A man leaving his office on one afternoon noticed the clock at past two o'clock. Between two three hours, he
returned to his office noticing the hands of the clock interchanged. At what time did he leave the office and the
time that he returned to the office?
a. 2:27.08, 5:11.19 P.M.
b. 2:26.01, 5:10.01 P.M
c. 2:26.01, 5:10.01 P.M.
d. 2:26.01, 5:12.17 P.M.
114. A medium unshaded lamp hangs 8 m directly above the table. To what distance should it be lowered to
increase the illumination to 4.45 times the former value? Illumination intensity varies inversely to the square of the
distance.
a. 4.75 m
b. 4.55 m
c. 3.79 m
d. 3.95 m

115. Roberto is 25 years younger than his father. However, his father will be twice his age in 10 years. Find their
ages now.
a. 15 and 40
b. 10 and 35
c. None of the choices
d. 20 and 45

116. A storage battery discharges at a rate which is proportional to the charge. If the charge is reduced by 50% of
its original value at the end of 2 days, how long will it take to reduce the charge to 25% of its original charge?
a. 6
b. 4
c. 3
d. 5

117. Prior to the last IBP elections, a survey was conducted in a certain barangay in Metro Manila to find out which
of three political parties they like best. The results indicated that 320 like KBL, 250 like LABAN and 180 liked
INDEPENDENTS. But of these, 160 like both KBL and LABAN, 100 liked both LABAN and INDEPENDENTS and 70 like
both KBL and INDEPENDENTS. Only 30 said they like all the three parties and none admitted that they did not like
any party. How many voters are there in the barangay?
a. 474
b. 525
c. 450
d. 540

118. A man left his home at past 3:00 o'clock P.M as indicated in his wall clock. Between 2 and 3 hours after, he
returned home and noticed the hands of the lock interchanged. At what time the man leave his home?
a. 3:24.73 P.M
b. 3:18.52 P.M
c. 3:31.47 P.M
d. 3:28.65 P.M

119. Given: f(x) = ( x+ 3) (x - 4) +4. When f(x) is divided by (x - k), the remainder is k. Find k.
a. 2
b. 6
c. 4
d. 8

120. A & B working together can finish painting the house in six days. A working alone, can finish it in five days less
than B. How long will it take each of them to finish the work alone?
a. 15 days for A
20 days for B
b. 10 days for A
25 days for B
c. 15 days for A
20 days for B
d. 10 days for A
15 days for B

121. A statistical clerk submitted the following reports: "The average rate of production of radios is 1.5 units for
every 1.5 hours of work by 1.5 workers." How many radios were produced in one month by 30 men working 200
hours during the month?
a. 4000
b. 3500
c. 4500
d. 5000

122. A piece of paper is 0.05 in thick. Each time the paper is folded into half, the thickness is doubled. If the paper
was folded 12 times, how thick in feet the folded paper will be?
a. 15.2
b. 16.25
c. 17.06
d. 18.5

123. A job could be done by eleven workers in 15 days. Five workers started the job. They were reinforced with
four more workers at the beginning of the 6 th day. Find the total number of days it took them to finish the job.
a. 22.36 days
b. 20.56 days
c. 23.22 days
d. 21.42 days

124. Six times the middle digit of a three-digit number is the sum of the other two. If the number is divided by the
sum of its digits, the answer is 51 and the remainder is 11. If the digits are reversed the number becomes smaller
by 198, find the number.
a. 825
b. 775
c. 725
d. 875

125. Given that "w" varies directly as the product of x and y and inversely as the square of z and that w = 4 when x
= 2, y = 6 and z = 3. Find tha value of "w" when x = 1, y = 4 and z = 2.
a. 5
b. 4
c. 3
d. 2

126. A man driving his car at a certain speed from his house will reach his office in 6 hours. If he increased his
speed 15 mph, he would reach his office 1 hour earlier. Find the distance from his office to his house.
a. 350 miles
b. 450 miles
c. 520 miles
d. 250 miles

127. Determine x, so that x, 2x + 7, 10x - 7 will be a geometric progression.


a. 7, -15/6
b. 7, -7/5
c. 7, -5/6
d. 7, -7/6
128. Solve for the values of x and y in 4x + 2y = 5 and 13x - 3y = 2.
a. (1, 3)
b. (3/2, 1/2)
c. (1, 2)
d. ( 1/2, 3/2 )

129. Determine the k so that the equation 4x2 + kx + 1 = 0 will have just one real root.
a. 5
b. 6
c. 4
d. 3

130. An airplane travels from points A and B with the distance of 1500 km and a wind along its flight line. If it takes
the airplane 2 hours from A to B with the tailwind and 2.5 hours from B to A with the headwind, what is the
velocity?
a. 700 kph
b. 675 kph
c. 450 kph
d. 750 kph

131. How many numbers between 10 and 200 are exactly divisible by 7? Find their sum.
a. 2835
b. 2840
c. 283
d. 2830
e. 27 numbers; sum
f. 28 numbers; sum
g. 26 numbers; sum
h. 26 numbers; sum

132. A gasoline tank of a car contains 50 liters of gasoline and alcohol, the alcohol comprising 25%. How much of
the mixture must be drawn off and replaced by alcohol so that the tank will contain a mixture of which 50% is
alcohol?
a. 10.67 liters
b. 20.33 liters
c. 16.67 liters
d. 16.33 liters

133. In a pile of logs, each layer contains one more log than the layer above and the top contains just one log. If
there are 105 logs in the pile, how many layers are there?
a. 16
b. 14
c. 10
d. 12

134. Two thousand (2000) kg of steel containing 8% nickel is to be made by mixing a steel containing 14% nickel
with anothercontaining 6% nickel. How much of each is needed?
a. 800 kg, 1200 kg
b. 500 kg, 1500 kg
c. 600 kg, 1500 kg
d. 400 kg, 1600 kg
135. A boat man rows to a place 4.8 miles with the stream and black in 14 hours, but that he can row 14 miles with
the stream in the same time as 3 miles against the stream. Find the rate of the stream.
a. 1 mile per hour
b. 0.6 mile per hour
c. 0.8 mile per hour
d. 1.5 mile per hour

136. Gravity causes a body to fall 16.1 ft in the first second, 48.3 ft in the 2 nd second, 80.5 ft in the 3rd second. How
far did the body fall during the 10th second.
a. 250.1 ft
b. 305.9 ft
c. 529.45 ft
d. 417.3 ft

137. Solve for x : 10x2 + 10 x2 + 1 = 0.


a. -0.331, 0.788
b. -0.311, -0.887
c. -0.113, -0.788
d. -0.113, -0.887

138. An airplane travels from points A and B with a distance of 1500 km and a wind along its flight line. If it takes
the airplane 2 hours from A and B with the tailwind and 2.5 hours from B to A with the headwind, What is the
velocity?
a. 700 kph
b. 675 kph
c. 750 kph
d. 450 kph

139. A jogger starts a course at a steady rate of 8 kph. Five minutes later, a second jogger starts the same course at
10 kph. How long will it take the second jogger to catch the first?
a. 22 min
b. 18 min
c. 21 min
d. 20 min

140. A rubber ball is made to fall from a height of 50 ft. and is observed to rebound 2/3 of the distance it falls. How
far will the ball travel before coming to rest if the ball continues to fall in this manner?
a. 300
b. 200
c. 350
d. 250

141. The resistance of the wire varies directly with its length and inversely with its area. If a certain piece of wire 10
m long and 0.10 cm in diameter has a resistance of 100 ohms, what will its resistance be if it is uniformly stretched
so that its length becomes 12 m?
a. 144
b. 80
c. 120
d. 90
142. Ten liters of 25% salt solution and 25 liters of 35% salt solution are poured into a drum originally containing 30
liters of 10% salt solution. What is the percent concentration of salt in the mixture?
a. 0.1955
b. 0.2572
c. 0.2215
d. 0.2705

143. A & B can do the job in 42 days, B & C for the same job in 31 days, C & A also for the same job in 20 days. If A
& C work together, how many days can they do the same job?
a. 19
b. 17
c. 21
d. 15

144. A pipe can fill a tank in 14 hours. A second pipe can fill the tank in 16 hours. If both pipes are left open,
determine the time required to fill the tank?
a. 7.92 hr
b. 8.47 hr
c. 7.47 hr
d. 6.53 hr

145. A man rows downstream at the rate of 5mph and upstream at the rate of 2mph. How far downstream should
he go if he is to return in 7/4 hours after leaving?
a. 2.5 miles
b. 3.3 miles
c. 2.7 mlies
d. 3.1 miles

146. Solve for the value of x.


2x - y + z = 6
x - 3y - 2z = 13
2x - 3y - 3z = 16
a. 3
b. 1
c. 2
d. 4

147. Find the value of w in the following equations:


3x - 2y + w = 11
x + 5y - 2w = -9
2x + y - 3w = -6.
a. 4
b. 2
c. 3
d. -2

148. A boat travels downstream 2/3 of the time as it goes going upstream. If the velocity of the river's current is 8
kph, determine the velocity of the boat in still water.
a. 70 kph
b. 60 kph
c. 30 kph
d. 40 kph
149. A survey of 100 persons revealed that 72 of them had eaten at restaurant P and that 52 of them had eaten at
restaurant Q. Which of the following could not be the number of persons in the surveyed group who had eaten at
both P and Q?
a. 23
b. 22
c. 24
d. 25

150. It takes an airplane one hour and forty-five minutes to travel 500 miles against the wind and covers the same
distance in one hour and fifteen minutes with the wind. What is the speed of the airplane?
a. 285.75 mph
b. 450.50 mph
c. 375.50 mph
d. 342.85 mph

Basic Engineering Correlation (Trigo Reviewer)

1. What will be the length of the two other sides of a right triangle if the opposite side of a 60 degrees angle is 4V
cm _____"
a. 8cm, 4 cm
b. 4 cm, 3 cm
c. 2 cm, 1cm
d. 4cm, 5 cm

2. The expression sin16° sin14° + cos16° cos14° is equivalent to


a. Cos 8°
b. Sin 30°
c. Sin 8°
d. Cos 2°

3. If tan a = 1/2 — and tan = -3/5, then the value of tan(a +,8) is
a. 5/9
b. 7/9
c. 9/7
d. 11/7

4. What is the value of sin [3 if cos p = 3/5?


a. sec 0 = 0.8
b. sine=0.25
c. cot 0=0.5
d. tan@=2.5

5. A central angle of 45 degrees subtends an arc of 12 cm. What is the radius of the circle?
a. 12.58 cm
b. 15.82 cm
c. 12.82 cm
d. 15.28 cm

6. The exact radian measure of 180o is


a. π
b. 3π
c. 4π
d. 2π

7. Solve for x by logarithm, log x2 - log (2x/5 = 7.58.


a. 189675888
b. 136783520
c. 15207576
d. 13678352

8. If arctan x + arctan (1/3) = π /4, the value of x is ______.


a. ½
b. 1/5
c. 1/3
d. ¼

9. A road is tangent to a circular lake. Along the road and 12 miles from the point of tangency, another road opens
towards the lake. From the intersection of the two roads to the periphery of the lake, the length of the new road is
11 miles. If the new road will be prolonged across the lake, find the length of the bridge to be constructed.
a. 2.09 miles
b. 6.65 miles
c. 1.20 miles
d. 8.89 miles

10. A strip of 640 sq. m is sold from a tirangular field whose sides are 96, 72 and 80 meters. The strip is of uniform
width "h" and has one of its sides parallel to the longest side of the field. Find the width of the strip.
a. 7.059 m
b. 5.89 m
c. 5.78 m
d. 6.679 m

11. The expression sin25x sin5x is equivalent to


a. 2sin10xcos5x
b. Sin20x
c. 2sin10xsin5x
d. 2sin15xsin10x

12. The area of the sector determined by an angle of 60° in a circle of radius 5 cm is
a. 49.09 cm2
b. 2812.5 cm2
c. 312.5 cm2
d. 13.09 cm2

13. Three times the sine of a certain angle is twice of the square of the cosine of the same angle. Find the angle.
a. 60o
b. 45o
c. 10o
d. 30o

14. If sin A = 2.5x and cos A = 5.5x, find the value of A in degrees.
a. 24.44
b. 32.47
c. 52.37
d. 42.47

15. One leg of a right triangle is 20 cm and the hypotenuse is 10 cm longer than the other leg. Find the length of
the hypotenuse.
a. 10 cm
b. 15 cm
c. 20 cm
d. 25 cm

16. Which of the following is an even function?


a. f(x)=3sin x
b. f (x)=4 tan x
c. f (x)=5 COSx
d. f (x)=4 cot x

17. The sides of a triangle lot are 130m, 180m, and 190m. This lot is to be divided by a line bisecting the longest
side and drawn from the opposite vertex. Find the length of the line (1) and the area of each lot (A).
a. 1125 m, A6520 sq.m
b. 1128 m, A2879 sq.m
c. 1110 m, A1000 sq. m
d. 1125 m, A5620 sq. m

18. If A is in the III quadrant and cos A = -15/17, find the value of cos (1/2)A.
a. -(8/17)1/2
b. -(2/17)1/2
c. -(1/17)1/2
d. -(5/17)1/2

19. Ship "A" started sailing N 40o 32' E at the rate of 3 mph. After 2 hours, ship "B" started from the same port
soing S 45o 18' E at the rate of 4 mph. After how many hours wil the second ship be exactly south of ship "A"?
a. 4.37 hours
b. 2.37 hours
c. 5.37 hours
d. 3.37 hours

20. Two speedboats simultaneously sailed out from port A on a 10 km radius circle lake towards point B directly
opposite of port A. The first boat took the shortest route and reached the destination in 1 hour. The boat has to
pass by port C before proceeding to port B. At what speed will it run in order to arrive at port B at the same time
with the first boat?
a. 78.89 kph
b. 67.89 kph
c. 34.57 kph
d. 27.32 kph

21. The reference angle of 0 = 210° is


a. 15°
b. 45°
c. 60°
d. 30°

22. If A is in the III quadrant and cos A = -15/17, find the value of cos (1/2)A.
a. -(2/17)1/2
b. -(8/17)1/2
c. -(5/17)1/2
d. -(1/17)1/2

23. The angle that is supplementary to 45o 15' 25" is


a. 45.257o
b. 44.743o
c. 134.74o
d. 44o 45'

24. If 77o + 0.40x = arc tan (cot 0.25x), find x.


a. 30o
b. 10o
c. 20o
d. 40o

25. If A + B + C = 180o and tan A + tan B + tan C = 5.67, find the value of tan A tan B tan C.
a. 1.89
b. 5.67
c. 1.78
d. 6.75

26. The angle of elevation of the top point D of a tower A is 23 o30'. From another point B the angle of elevation of
the top of the tower is 55o30'. The points A and B are 217.45 m. apart and on the same horizontal plane as the foot
(point C) of the tower. The horizontal angle subtended by A and B at the foot of the tower is 90 o. Find the height of
the tower CD.
a. 69.59 m
b. 90.59 m
c. 59.90 m
d. 50.90 m

28. The simplified form of sin4 0 —cos4 0 is


a. 0
b. 1
c. 2sin2 0-1
d. 1+2cos2

29. The simplified form of cos(A— B)—cos(A + B) is


a. Cos2B
b. math012-1t-
c. Cos2A
d. 2sinAsinB

30. If cot 2A cot 68o = 1, then tan A is equal to _______.


a. 0.194
b. 0.491
c. 0.491
d. 0.419

31. The exact degree measure of 0.5π is


a. 45o
b. 90o
c. 180o
d. 145o

32. Solve for G if csc (11G - 16o) = sec (5G + 26o).


a. 5 degrees
b. 6 degrees
c. 4 degrees
d. 7 degrees

33. A ladder 5 m long leans against the wall of an apartment house forming an angle of 50 degrees, 32 minutes
with the ground. How high od the wall does it reach?
a. 3.12 m
b. 2.00 m
c. 12.66 m
d. 3.86 m

34. A regular dodecagon is inscribed in a circle of radius 24. Find the perimeter of the dodecagon.
a. 151.24
b. 153.25
c. 143.63
d. 149.08

35. The measure of 2.25 revolutions counterclockwise is


a. 810 degrees
b. 835 degrees
c. 810 degrees
d. 805 degrees

36. Determine the amplitude and the phase shift for the function f(t)= 2 sin (3x + 4)
a. 2 and 4/3
b. 2 and -4/3
c. 2 and ¾
d. 2 and -3/4

37. Solve angle A of an oblique triangle with vertices ABC, if a = 25, b = 16 and C = 94 degrees and 6 minutes.
a. 49 degrees and 37 minutes
b. 55 degrees and 32 minutes
c. 53 degrees and 40 minutes
d. 54 degrees and 30 minutes

38. The terminal side of the angle θ = 500 o in standard position is in quadrant.
a. III
b. IV
c. I
d. II

39. Determine the period of the curve y = sin(1/2)x


a. 540o
b. 360o
c. 900o
d. 720o

40. Solve for x in the given equation: arc tan(x +1) +arc tan (x - 1) = arc tan (12).
a. 1.5
b. 1.2
c. 1.34
d. 1.25

41. Two towers AB and CD are of equal height. At a point between them in the line AC joining their bases, the
angle of elevation of the nearer tower was observed to be 60 o. Then at 24 m from the same point in a direction
perpendicular to AC, the angle of elevation of the top of the towers are 45 o for the nearer tower and 30o for the
other. Find the height of the towers (h) and their distance apart (x).
a. h=29.38 m, x=71.83 m
b. h=39.38m, x=61.83 m
c. h=49.83, x=61.83 m
d. h=29.38 m, x=61.83 m

42. If 3x = 9y and 27y = 81z, find x/z.


a. 4/3
b. 8/3
c. 3/8
d. 3/5

43. Which of the following is a co terminal angle of  θ = 265o?


a. –95o
b. 615o
c. 585o
d. 65o

44. The terminal side of 0 if cote > 0 and sec() >0 is in quadrant
a. III
b. I
c. II
d. IV

45. Given: x = (cos B tan B - sin B) / cos B. Solve for x if B = D45 degrees.
a. 0.5
b. 0.577
c. 0.866
d. 0

46. The perimeter of an isosceles right triangle is 6.6824. Its area is


a. ½
b. 4
c. 2
d. 1

47. Simplify: 4 cos y sin y (1 - 2 sin 2y)


a. sec 4y
b. tan 4y
c. cos 4y
d. sin 4y

48. The angle of elevation of the top of the tower A from the foot of tower B is twice the angle of elevation of the
top of tower B from the foot of tower A. At a point midway between the two towers, the angles of elevations of
the top of the towers are complimetary. If the two towers are 120 m apart, what are the heights of the towers?
a. 30 m and 50 m
b. 30 m and 40 m
c. 25 m and 35 m
d. 40 m and 90 m

49. Find the value of x in the equation csc x + cot x = 3.


a. π /2
b. π /4
c. π /3
d. π /5

50. Find the other parts of the triangle given a = 48°,1C = 57 degrees b = 47 units.
a. 75 °, 36.16 units
b. 75 °, 35.16 units
c. 75 °, 33.16 units
d. 75 °, 34.16 units

51. A clock has a dial face 12 inches in radius. The minute hand is 9 inches long while the hour hand is 6 inches
long. The plane of rotation of the minute hand is 2 inches above the plane of rotation of the hour hand. Find the
distance between the tips of the hands of the clock at 5:40 a.m.
a. 3.89 in
b. 8.67 in
c. 7.78 in
d. 9.17 in

52. The expression 2cos6x cos2x is equivalent to


a. cos10x + cos6x
b. cos5x + cos3x
c. Cos8x + cos4x
d. cos32x

53. The solution set of the equation(tan x) 2 — 1 = 0 on the interval [0°, 360°) is ~{30°,90°,150°1}
a. {45°,135°,225°,315°1
b. {0°,30°,330°
c. 160901

54. If the terminal side of angle 13 contains the point (-5, -7) then 13 is equal to
a. — 35.54°
b. 35.54°
c. 234.46°
d. 54.47°

55. Simplify the expression: (sin B + cos B tan B) / cos B.


a. tan B cos B
b. tan B + cos B
c. 2 sin B cos B
d. 2 tan B

56. A 40 m high tower stands vertically on a hillside (sloping ground) which makes an angle of 18 o with the
horizontal. A tree also stands vertically up the hill from the tower. An observer on top of the tower finds the angle
of depression of the top of the tree to be 26o and the bottom of the tree to be 38o. Find the height of the tree.
a. 59.89 m
b. 89.89 m
c. 35.67 m
d. 10.62 m

57. Triangle ABC is a right triangle with the right angle at C. CD is perpendicular to AB. BC = 4, and CD = 1. Find the
area of the triangle ABC.
a. 2.7
b. 2.07
c. 2.11
d. 2.43

58. If sin A = 4/5, A is in quadrant II, sin B = 7/25, B is in quadrant I. Find sin (A + B).
a. 2/5
b. ¾
c. 3/5
d. 4/5

59. A and B are summit of two mountains rise from a horizontal plain, B being 1200 m above the plain. Find the
height of A, it being given that its angle of elevation as seen from a point C in the plain (in the same vertical plane
with A and B) is 50o, while the angle of depression of C viewed from B is 28 o58' and the angle subtended at B by AC
is 50o.
a. 2890.89 m
b. 1002.33 m
c. 1309.90 m
d. 3002.33 m

60. 174 degrees is equivalent to _____ mils.


a. 2044
b. 2845
c. 3421
d. 3094

61. Which of the following is arccos(n)?


a. UNDEFINED
b. n
c. 0
d. 1

62. A cyclic quadrilateral has the sides AB = 8 cm; and CD = 12 cm. The fourth side DA forms the diameter of the
circle. Find the area of the circle.
a. 467.89 sq. cm
b. 87.89 sq. cm
c. 657.89 sq. cm
d. 316.68 sq. cm

63. If tan 25 = m, find the value of tan (tan 155 - tan 115) / (1 + tan 115 x tan 155). ( Note: all angles are in degrees).
a. (m2 + 1) / 2m
b. m2 + 1
c. (1 - m2) / 2m
d. (m2 - 1) / 2m

Basic Engineering Correlation (Analytic Geometry Reviewer)


 
1. The graph of the polar equation: r = 2cos0 is a
a. Rose
b. Limacon with a loop
c. Circle
d. Cardioid
 
2. Classify the conic represented by the equation x2 + 4xy + 5 y2 - x + 2y + 1 = 0
a. circle
b. ellipse
c. hyperbola
d. parabola
 
3. The graph of the polar equation: r = I l is
a. a circle
b. a parabola
c. an ellipse
d. a hyperbola
 
4. What is the slope of the line 4x-5y +6 = 0?
a. -5/4
b. 5/4
c. 4/5
d. -4/5
 
5. The point of intersection of the lines x — 2y+4 0 and —3x + y —2 =0 is
a. (0,2)
b. (-2,0)
c. (0,-2)
d. (2,0)
 
6. The conic given by the equation? +4xy+5y2 -x+ 2y+1 =0 is
a. parabola
b. circle
c. ellipse
d. hyperbola
 
7. Find the slope of a line having a parametric equation of y = 4t + 6 and x = t + 1.
a. 2
b. 1
c. 4
d. 3
 
8. Find the equation of a straight line with a slope of 3 and a y-intercept of 1.
a. x + 3y + 1
b. 0
c. x - 3y - 1
d. 3x - y + 1
e. 3x + y - 1
f. 0
g. 0
h. 0
 
9. An equation of the line with x and y intercepts 7 and -7, respectively, is
a. x— y +7 =0
b. x —y-7 = 0
c. —x+y-7=0
d. x+y+7=0
 
10. The line joining the points (3, -1) and (-3, 2) has equation
a. x+2y+1= 0
b. x+2y—l= 0
c. x+y-2=0
d. x-2y—I=0
 
11. The set of all points in a plane such that the sum of the distances of a point from some fixed points on the
plane is a constant is a/an
a. parabola
b. ellipse
c. hyperbola
d. circle
 
12. The distance from the point (5, 2) to the line 8x - 6y +2 = 0 is
a. 3
b. 4
c. 1
d. 2
 
13. If (3,-5) is the midpoint of (-1,-3) and (x, y), then the values of x and y are
a. x=1, y=-4
b. x=7, y= -7
c. x=2, y= -1
d. x= 7, y= -1
 
14. The distance between the given lines 3x + 2y —2 =0 and 3x +2y-6 =0 is
a. 1.39
b. 1.12
c. 0.28
d. 0.55
 
15. The major axis of the elliptical path in which the earth moves around the sun is approximately 186,000,000
miles and the eccentricity of the ellipse is 1/60. Determine the apogee of the earth.
a. 94,550,000 miles
b. 93,000,000 miles
c. 91,450,000 milse
d. 94,335,100 miles
 
16. A line 4x + 2y -2 = 0 is coincident with the line
a. 0
b. 0
c. 4x + 4y + 2
d. 4x + 3y + 3
e. 8x + 4y - 2
f. 0
g. 0
h. 8x + 4y - 4
 
17. The length of the semi-transverse axis of the graph of --- 9 — 4 =1 is
a. 2
b. 3
c. 5
d. 4
 
18. The equation of the line through (1, 2) and perpendicular to 6x - y +5 =0 is
a. 6x-y-11=0
b. 6x-y+5=0
c. x+6y-13=0
d. x+6y-8=0
 
19. If the distance between the points (h, 2) and (0, 4) is 2 then the value of h is
a. 3,J2
b. 0
c. 2,5
d. 2
 
20. The length of the latus rectum for the ellipse 16x2 + 25y2 = 400 is equal to1
a. 5
b. 4
c. 6.4
d. 12.5
 
21. The graph of the polar equation: r = 2 + 2cos9 is a
a. limacon
b. Circle
c. cardioid
d. lemniscates
 
22. Find the angle formed by the lines 2x + y — 8 = 0 and x + 3y + 4 = 0
a. 30°
b. 60°
c. 45°
d. 35°
 
23. The equation of a line that intercepts the x-axis at x = 4 and the y-axis at y = -6 is,
a. 3x + 2y
b. 12
c. 2x - 3y
d. 12
e. 2x - 3y
f. 3x - 2y
g. 12
h. 12
 
24. Find the distance between the lines 3x + y - 12 = 0 and 3x + y - 4 = 0
a. letter d) 8/the squareroot of 10
 
25. Find the polar coordinate of the point (-3,A/3 )
a. (J18, 60°)
b. (412, 30°)
c. (012, 150°)
d. (A118, 330°)
 
26. To simplify the equation x2 + 4y2 + 6x +16y + 21= 0 by translation of axes, the origin must be moved to
a. (-3, -2)
b. (2, 3)
c. (3, 8)
d. (-3, -8)
 
27. Given the equation of the parabola x2 = 4y – 20 . Locate its vertex.
a. (4, 20)
b. (0, 5)
c. (0, 4)
d. (0, 20)
 
28. Find the equation of a straight line with a slope of 1/2 and y-intercept 3.
a. x - 2y -3 = 0
b. 3x-y+2 =0
c. x-2y+ 6 =
d. 2x-y+3 =0
 
29. Determine the coordinates of the point which is two-fifths of the way from the point (1,-5) to the point (6,10)
a. (3, 1)
b. (4, 5)
c. (2, -2)
d. (3, 5)
 
30. Find the area of the circle whose equation is x2 + y2 = 6x - 8y.
a. 25 &
b. 20 &
c. 30 &
d. 15 &
 
31. How far from the y-axis is the center of the curve 2x2 +2y2 + 10x - 6y - 55 = 0?
a. -3.25
b. -3.0
c. -2.5
d. -2.75
 
32. Which of the following lines is parallel to the line 6x — 4y = 7?
a. 6x + 4y = 6
b. 4x — 6y = 9
c. 3x - 2y = 15
d. 3x + 2y = 12
 
33. The slope of the line passing through (-2,2) and (3,12).
a. -2
b. ½
c. 2
d. 10
 
34. A line 4x + 2y -2 = 0 is coincident with the line
a. 0
b. 4x + 3y + 3
c. 0
d. 8x + 4y - 2
e. 8x + 4y - 4
f. 0
g. 0
h. 4x + 4y + 2
 
35. The parabolic antenna has an eqaution of y2 + 8x = 0. Determine the length of the latus rectum.
a. 8
b. 12
c. 10
d. 9
 
36. 14. A line through (-5, 2) and (1, -4) is perpendicular to the line through (x, -7) and (8, 7). Find x.
a. -4
b. -19/3
c. -6
d. -5
 
37. Find the eccentricity of the curve 9x2 - 4y2 - 36x + 8y = 4
a. 1.92
b. 1.86
c. 1.8
d. 1.76
 
38. If the points (0,0), (2, 0), and (1, k) are vertices of an equilateral triangle then a value of k is
a. I
b. 5
c. 0
d. 2
 
39. Find the inclination of the line passing through (-5, 3) and (10, 7).
a. 14.63
b. 14.73
c. 14.83
d. 14.93
 
40. What is the equation of the line that passes thru (4, 0) and is parallel to the line x - y - 2 = 0
a. x - y
b. 0
c. x + y + 4
d. 0
e. x - y - 4
f. 0
g. 0
h. x - y + 4
 
41. What are the coordinates of the center of the curve x2 + y2 - 2x - 4y - 31 = 0
a. (2, 1)
b. (-1, -1)
c. (1, 2)
d. (3, 5)
 
42. If a line through (-5, 2) and (1, -4) is parallel to the line through (x, -7) and (8, 7) then x =
a. -5
b. -6
c. 22
d. -4
 
43. Find the distance between the given lines 4x - 3y = 23 and 4x - 3y = -7
a. 3
b. 4
c. 6
d. 5
 
44. The equation of the directrix of the parabola y2 = 20x is
a. x = -5
b. x = 5
c. x = 4
d. x = -4
 
45. The center of a circle is at (1, 1) and one point on its circumference is (-1, -3). Find the other end of the
diameter through ( -1, -3).
a. (3, 6)
b. (2, 4)
c. (1, 3)
d. (3, 5)
 
46. Two vertices of a triangle are (2, 4) and (-2, 3) and the area is 2 square units, the locus of the third vertex is
a. x + 4y = 12
b. 4x - y =14
c. 4x + 4y = 14
d. x - 4y =-10
 
47. The focus of parabola y2 = 16x is at:
a. (0, 3)
b. (3, 0)
c. (0, 4)
d. (4, 0)
 
48. The diameter of a circle described by 9x2 + 9y2 = 16 is
a. 4/3
b. 16/9
c. 4
d. 8/3
 
49. Find the distance between the points A (4, 7) and B (-1, -5).
a. 10
b. 5
c. 13
d. 12
 
50. The equation 25x2 + 16y2 - 150x + 128y + 81 = 0 has its center at
a. (3, -4)
b. (3, 5)
c. (3, 4)
d. (4, -3)
 
51. Find the equation of the line where x-intercept is 2 and y-intercept is -2.
a. x - y - 2
b. 0
c. 2x + 2y +2
d. -2
e. 0
f. -2x +2y
g. 0
h. x - y - 1
 
52. Find the inclination of the line passing through (-2,4) and (2,7)
a. 53.13
b. 90
c. 36.87
d. 70
 
53. A horizontal line has a slope of
a. zero
b. infinity
c. negative
d. possitive
 
54. To simplify the equation x2 + 4y2 + 6x +16y+ 21= 0 by translation of axes, the origin must be moved to
a. (-3, -2)
b. (-3, -8)
c. (3, 8)
d. (2, 3)
 
55. Find the eccentricity of the curve 9x2 –16y2 –144 = 0
a. 0.66
b. 1.67
c. 0.88
d. 1.25
 
56. Find the eccentricity of the curve 9x2 —16y2 —144= 0
a. 1.67
b. 0.66
c. 1.25
d. 0.88
 
57. In the equation y = x2 + x + 1, where is the curve facing?
a. Downward
b. Facing left
c. Facing right
d. Upward
 
58. Find the acute angle of rotation such that the transformed equation of 6x2 +31y + 4y2 + x y = 0 will have no x' y'
term.
a. 16.85°
b. 28.15°
c. 53.13°
d. 53.13°
 
59. The equation of the line through (1, 2) and perpendicular to 6x + y — 4 = 0 is
a. x+6y-4 =0
b. x+2y-4 =0
c. 6x+y+ 4 =0
d. x- 6y+ 11 =
 
60. The equation of the line with a slope 47 and y-intercept -2 is
a. —4 5x—y+2=0
b. x+y-2 =0
c. 5x-4y-20=0
d. 4x-5y-20=0
 
61. The polar equation r = 1 when transformed into a rectangular equation is cos° —4sin
a. x2 — 4y2 =I
b. 4x2 — y2 =1
c. 4x — y = 4
d. x —4y = I
 
62. Given three vertices of a triangle whose vertices are A(1, 1), B(3, -3) and (5, -3). Find the area of the triangle.
a. 6 sq. units
b. 5 sq. units
c. 4 sq. units
d. 3 sq. units
 
63. A line with slope equal to — 2 has an inclination of
a. 116.57°
b. —116.57°
c. 63.43°
d. —63.43°
 
64. What is the distance between the centers of the circle x2 + y2 + 2x + 4y - 3 = 0 and x2 + y2 - 8x - 6y + 7 = 0?
a. 7.07
b. 7.77
c. 8.07
d. 7.87
 
65. The area of hexagon ABCDEF formed by joining the points A(1, 4), B(0, -3), C(2, 3), D(-1, 2), E(-2, -1) and F(3, 0)
is _________ square units.
a. 15
b. 24
c. 22
d. 20
 
66. Determine B such that 3x + 2y -7 = 0 is perpendicular to 2x - By + 2 = 0.
a. 4
b. 2
c. 5
d. 3
 
67. Find the distance between the A (4, -3) and B (-2, 5).
a. 10
b. 8
c. 11
d. 9
 
68. The equation of a line that intercepts the x-axis at x = 5 and the y-axis at y = -4 is
a. 88x- l0y = 40
b. 5x + 4y = 20
c. 10x - 8y = 20
d. 4x + 5y = 20
 
69. Find the value of k for which the equation x2 + y2 + 4x - 2y - k = 0 represents a point circle.
a. 6
b. 5
c. -6
d. -5
 
70. What is the length of the latus rectum of the curve x2 = 20y
a. 5
b. 20
c. √ 20
d. √ 5
 
71. Find the acute angle of rotation such that the transformed equation of 6x2 + 3xy+ 4y2 +x-y =0 will have no x' y'
term.
a. 16.85°
b. 28.15°
c. 36.86°
d. 53.13°
 
72. Find the coordinates of the point P(2, 4) with respect to the translated axis with origin at (1,3).
a. (1, 1)
b. (-1, 1)
c. (1, -1)
d. (-1, -1)
 
73. Determine the equation of the circle whose radius is 5, center on the line x = 2 and tangent to the line 3x - 4y +
11 = 0.
a. (x - 2)2 + (y - 2)2=25
b. (x - 2)2 + (y - 2)2=5
c. (x - 2)2 + (y + 2)2=25
d. (x - 2)2 + (y + 2)2=5
 
74. The equation x2 + y1- 8x – 2y + 1 = 0 describes
a. A. a circle of radius 4 centered at (4, 1)
b. a circle of radius 4 centered at (-4,-1)
c. an ellipse centered at (-4, -1)
d. an ellipse centered at (4, 1)
 
75. Find the equation of a circle whose center is at (3, -5) and whose raduis is 4.
a. (x - 3)2 + (y + 5)2
b. (x - 5)2 + (y - 3)2
c. 16
d. 16
e. (x
f. 3)2 + (y - 5)2
g. 16
h. 16
i. x2 + y2
 
76. The line passing through the focus and is perpendicular to the directrix of the parabola.
a. axis of the parabola
b. latus rectum
c. directrix
d. tangent line
 
77. What is the equation of the line joining the points (3, -2) and (-7, 6)?
a. 2x + 3y = 0
b. 4x + 5y - 0
c. 5x + 4y = 7
d. 4x - Sy 22= 0
 
78. The angle formed by the lines y = -2x +8 and y =1x- -4 is
a. 45°
b. 35°
c. 60°
d. 30°

79. In general quadratic equation, if the discriminant is zero, the curve is a figure that represents a/an _______.
a. circle
b. hyperbola
c. ellipse
d. parabola

80. The directrix of a parabola is the line y = 5 and its focus is at the point (4, -3). What is the length of the latus
rectum?
a. 18
b. 12
c. 14
d. 16

81. A line, which is perpendicular to the x-axis, has a slope to


a. infinity
b. 1
c. -1
d. 0
82. A line passes thru (1, -3) and (-4, 2). Write the equation of the line in slope-intercept form.
a. y - 4 - x
b. y - 2 - x
c. y- x - 2
d. y - x -4

83. The line segment connecting (x, 6) and (9, y) is bisected by the point (7, 3). Find the values of x and y.
a. 14, 6
b. 5, 0
c. 33, 12
d. 14, 3

84. Determine the coordinates of the point which is three-fifths of the way from the point (2, -5) to the point (-3,
5).
a. (1, -1)
b. (-1, 1)
c. (-1, -2)
d. (-2, -1)

85. Which of the following points lie on the fourth quadrant?


a. (5, 57r/4)
b. (-4, 27r13)
c. (-4, -7rJ3)
d. (5, -77r16)

86. The midpoint of the line segment between P1(x1, y1) and p2(-2, 4) is P(2, -1). Find the coordinates of P1.
a. (-6, 6)
b. (6, -6)
c. (5, -6)
d. (6, 6)

87. A locus of a point which moves so that it is always equidistat from a fixed point (focus ) to a fixed line (directix)
is a _______.
a. hyperbola
b. ellipse
c. circle
d. parabola

88. A parabola having a span of 30m and a height of 20m has an area of
a. 540
b. 360
c. 400
d. 180

89. An equation of the line that is parallel to 3x-6y = —land passes through the point (2, 2) is
a. 2x—y+2=0
b. x-2y-2=0
c. x-2y+2 =0
d. x+2y+2= 0

90. If the product of the slope of any two straight line is negative 1, one of these lines are said to be
a. Skew
b. Non-intersecting
c. Parallel
d. Perpendicular

91. Find the slope of the line defined by y - x = 5.


a. -1/2
b. ¼
c. 1
d. 5 + x

Basic Engineering Correlation (Calculus Reviewer)

1. The depth of water in cylindrical tank 4 m in diameter is increasing at the rate of 0.7 m/min. Find the rate at
which the water flows into the tank.

a. 6.4

b. 2.5

c. 1.5

d. 8.8

2. The volume of the sphere is increasing at the rate of 6 cm 3 / hr. At what is its surface area increasing (in cn 2/hr)
when the radius is 50cm?

a. 0.3

b. 0.24

c. 0.4

d. 0.5

3. Find the height of aright circular cylinder of maximum volume, which can be inscribed in a sphere of radius 10
cm.

a. 12.81 cm.

b. 11.55 cm.

c. 15.11 cm.

d. 14.12 cm.
4. find the area in the first quadrant bounded by the parabola y2 = 4x, x = 1 and x = 3

a. 9.955

b. 5.955

c. 5.595

d. 9.555

5. Find the maximum point of y = x + 1/x

a. (1,2)

b. (2,3)

c. (-1, -2)

d. (2, 5/2)

6. ___________ is the concept of finding the derivative of composite functions.

a. Logarithmic differentiation

b. Implicit differentiation

c. Trigonometric differentiation

d. Chain Rule

7. Find the area bounded by the curve defined by the equation x2 = 8y and its latus rectum.

a. 22/3

b. 32/3

c. 16/3

d. 11/3

8. If y = x lnx. Find

a. -1/x
b. 1/x

c. -1/x2

d. 1/x2

9. Car A moves due east at 30 kph, at the same instant car B is moving S 30 o E with the speed 60 kph. The distance
from A to B is 30 km. Find how fast is the distance between them separating after 1 hour

a. 38 kph

b. 36 kph

c. 45 kph

d. 40 kph

10. A box is to be constructed from a piece of zinc 20 sq. in. by cutting equal squarea from each corner and turning
up the zinc to form the side. What is the volume of the largest box that can be so constructed?

a. 592.59 cu. in.

b. 622.49 cu. In

c. 579.50 cu. In

d. 599.95 cu. in.

11. Find the coordinates of the vertex of the parabola y = x2 - 4x + 1 by making use of the fact that at the vertex,
the slope of the tangent is zero.

a. (-2, -3)

b. (3, -2)

c. (-1, -3)

d. (2, -3)

12. Given the function f(x) = x3 - 6x +2. Fnd the first derivative at x = 2

a. 3x2 - 5
b. 8

c. 6

d. 7

13. If the first derivative of the function is constant, then the function is__________.

a. Linear

b. Logarithmic

c. Sinusoid

d. Exponential

14. Using the two existing corner sides of an existing wall, what is the maximum rectangular area that can be
fenced by a fencing material 30 ft. long?

a. 250 sq. ft.

b. 225 sq.ft.

c. 200 sq. ft.

d. 216 sq. ft.

15. The velocity of a body is given by v(t) = sin(xt), where the velocity is given in meters per second and " t " is given
in seconds. The distance covered in meters between t =1/4 and 1/2 second is close to

a. 0.5221 m

b. -0.5221 m

c. -0.2251 m

d. 0.2551 m

16. Differentiate y = ex cos x2

a. ex(cosx2 - 2x sinx2)

b. -2xex sinx2

c. -ex sinx2
d. ex cosx2 - 2x sinx2

17. Three sides of a trapezoid are each 8 cm. long. How long is the fourth side when the area of the trapezoid has
the greatest value?

a. 10

b. 8

c. 16

d. 12

18. Differentiate y = sec(x2 + 2)

a. -cos(x2 + 2)cot(x2 + 2)

b. 2xcos(x2 + 2)

c. cos(x2 + 2)

d. 2xsec(x2 + 2)tan(x2 + 2)

19. A statue 3 m high is standing on a base of 4 m high. If an observer's eye is 1.5 m above the ground, how far
should he stand from the base in order that the angle subtended by the statue is a maximum.

a. 3.41 m

b. 4.41 m

c. 3.51 m

d. 3.71 m

20. What is the area of the largest rectangle that can be inscribed in a semi-circle of radius 10?

a. 2 √ 50

b. 100

c. 1000

d. √ 50
21. Find the partial derivative with recpect to x of the funcyion xy2 - 5y + 6

a. 2xy

b. xy - 5y

c. y2 - 5

d. y2

22. In the curve 2 + 12x - x3, find the critical points.

a. (-2,18) & (2, -14)

b. (-2,18) & (-2,14)

c. (2,18) & (2,-14)

d. (2,18) & (-2,-14)

23. A man on a wharf 3.6 m above sea level is pulling a rope tied to a raft at 0.60 m/sec. How fast is the raft
approaching the wharf when there are 6 m of rope out?

a. -0.95 m/s

b. -0.75 m/sec

c. -0.65 m/sec

d. -0.85m/sec

24. Find of y = 3sin 2x

a. 3 cos 4x

b. 2 sin 2x

c. 6 cos x

d. 6 cos 2x

25. If the distance x from the point of departure at a time t is defined by the equation x = -16t 2 + 5000t + 5000,
what is the initial velocity?

a. 2000
b. 5000

c. 0

d. 3000

26. Find the slope of the tangent to the curve x2 + y2 - 6x + 10y + 5 = 0 at the point (1,0)

a. ¼

b. 2/5

c. 2

d. 1/5

27. Differentiate y = arc sin cos x

a. -2

b. 1

c. 2

d. -1

28. Evaluate the limit lnx/x as x approaches positive infinity.

a. 0

b. -1

c. 1

d. infinity

29. Determine the diameter of a closed a closed cylindrical tank having a volume of 11.3 cu. m. to obtain minimum
surface area.

a. 1.22

b. 2.68

c. 1.64
d. 2.44

30. Divide the number 120 into two parts such that the product of one and the square of the other is maximum.

a. 30 and 90

b. 60 and 60

c. 40 and 80

d. 50 and 70

31. Evaluate: Lim (2 - x)tan

a. b e

b. e2π

c. ∞

d. 0

32. Water is running into a hemispherical bowl having a radius of 10 cm. at a constant rate of 3 cu. cm/ min. When
the water is x cm. deep, the water level is rising at the rate of 0.0149 cm./min. What is the value of x?

a. 2

b. 4

c. 3

d. 5

33. Find the area bounded by the line x - 2y + 10 = 0, the x-axis, the y-axis and x = 10

a. 50

b. 75

c. 100

d. 25
34. Find the area bounded by the y - axis and x = 4 = y2/3

a. 12.8

b. 25.6

c. 56.8

d. 30.6

35. A triangle has variable sides x, y, z subject to the constaint such that the perimeter is fixed to 18 cm. What is
the maximum possible area for the triangle?

a. 14.03 sq.cm.

b. 15.59 sq. cm.

c. 17.15 sq. cm.

d. 18.71 sq. cm.

36. The cost of a product is a function of the quantity x of the product: C(x) = x2 - 400x + 50. Find the quantity for
which the cost is minimum.

a. 2000

b. 3000

c. 5000

d. 0

37. Find the slope of the line tangent to the curve y = x3 - 2x + 1 at x = 1.

a. 1/3

b. 1

c. 1/4

d. 1/2

38. Water is running out in a conical funnel at the rate of 1 cu. In. per second. If the radius of the base of the funnel
is 4 inches and the altitude in 8 inches, find the rate at which the water level is dropping when it is 2 inches from
top.
a. in./sec

b. in./sec

c. -1/9πin./sec.

d. in./sec

39. What is the area between y = 0, y = 3x2, x = 0 and x = 2?

a. 24

b. 6

c. 8

d. 12

40. If y = (t2 + 2)2 and t = x1/2, datermine

a. x5/2 + x1/2

b. 2(x + 2)

c. 3/2

d. letter b

41. Find the area between the curve y = cosh x and the x-axis from x = 0 and x = 1

Select one:

a. 1.667 sq. units

b. 1.333 sq. units

c. 1.125 sq.units

d. 1.175 sq. units

42. Find the second derivative of y by implicit differentiation from the equation 4x 2 + 8y2 = 36.

a. -9/4y3

b. -16/9y3
c. 32xy

d. 64x2

43. Find the area in sq. units bounded by the parabolas x2 - 2y = 0 and x2 + 2y - 8 = 0

a. 9.7

b. 4.7

c. 10.7

d. 11.7

44. What is the second derivative of a function y = 5x3 + 2x + 1?

a. 30x

b. 18

c. 30

d. 25x

45. Evaluate the limit of lim(x2 + 3x - 4) as x approaches 3.

a. 54

b. 14

c. 18

d. 72

46. The rate of change of function y with respect to x equals 2-y and y = 8 when x = 0. Find y when x = ln2

a. -2

b. -5

c. 2

d. 5
47. If y = 4 cos x + sin 2x, what is the slope of the curve when x = 2 radians?

a. -4.94

b. -2.21

c. 2.21

d. -3.25

48. Differentiate y = log10(x2 + 1)2   

a. 4x(x2 + 1)

b. log e(x)(x2 + 1)

c. None of the choices


None of the choices

d. 2x(x2 + 1)

49. Given a cone of diameter x and altitude of h. What percent is the volume of the largest cylinder which can be
inscribed in the cone to the volume of the cone?

a. 2.12

b. 2.25

c. 2.86

d. 2.51

50. Find the minimum distance from the point (4,2) to the parabola y 2 = 8x

a. 4 √ 3

b. 2 √ 3

c. √ 3

d. 2 √ 2

51. Find the area enclosed y the curve x2 + 8y + 16 = 0, the x - axis, the y-axis and the line x - 4 = 0

a. 8.67 sq. units


b. 9.67sq. units

c. 10.67 sq. units

d. 7.67 sq. units

52. Find the equation of the normal to x2 + y2 = 1 at the point (2,1).

a. 2x +3y = 3

b. y = 2x

c. x + y = 1

d. x = 2y

53. A poster is to contain 300 cm. sq. of printed matter with margins of 10 cm. at the top and bottom and 5 cm at
each side. Find the overall dimensions if the total area of the poster is minimum.

a. 22.24, 44.5

b. 27.76, 47.8

c. 25.55, 46.7

d. 20.45, 35.6

54. Find the equation of the normal to i>x2 + y2 = 5 at the point (2, 1)

a. x = 2y

b. x + y = 1

c. 2x +3y = 3

d. y = 2x

55. Find the equation of the curve at every point of which the tangent line has a slope of 2x.

a. y = -x2 + C

b. y = x2 + C

c. x = -y2 + C
d. 1x = y2 + C

56. The radius of spheres is r inches at time t seconds. Find the radius when the rates of increase of the surface
area and the radius are numerically equal.

a. 2π in

b. 1/4π in

c. π2 in

d. 1/8π in

57. Given a cone of diameter x and altitude of h. What percent is the volume of the largest cylinder which can be
inscribed in the cone to the volume of the cone?

a. 0.56

b. 0.44

c. 0.65

d. 0.46

58. The area enclosed by the ellipse (image) is revolved about the line x = 3. What is the volume generated?

a. 365.1

b. 360.1

c. 370.3

d. 355.3

59. If y = 2x + sin 2x, find x if y' = 0

a. π/2

b. 3π/2

c. π/4

d. 2π/3
60. A Norman window is in the shape of a rectangle surmountedby a semi-circle. What is the ratio of the width of
the rectangle to the total height so that it will yield a window admitting the most light for a given perimeter?

a. 1

b. 2/3

c. 2

d. ½

61. Find the area bounded by the parabola, x2 = 4y, and y = 4.

a. 33.21

b. 21.33

c. 13.23

d. 31.32

62. The area bounded by the curve y = 2x1/2, the line y = 6 and the y-axis is to be revolved at y = 6. Determine the
centroid of the volume generated.

a. 1.24

b. 0.56

c. 1.8

d. 1.0

63. Find the volume generated if the area between y = cosh x and x - axis from x = 0 to x = 1 is is revolved about the
x - axis.

a. 3.43 cu. Units

b. 4.42 cu. Units

c. 3.83 cu. Units

d. 2.83 cu. Units

64. What is the area bounded by the curve y = x3, the x-axis and the line x = -2 and x = 1?
a. 5.24

b. 2.45

c. 5.42

d. 4.25

65. Find the approximate increase by the use of differentials, in the volume of the sphere if the radius increases
from 2 to 2.05 in one second.

a. 2.12

b. 2.51

c. 2.86

d. 2.25

66. The integral of cos x wuth respect to x is

a. csc x + C

b. sec x + C

c. -sin x + C

d. sin x + C

67. Evaluate: Lim

a. infinity

b. 1

c. 0

d. 2

68. The distance of a body travels is a function of time t and is defined by: x(t) = 18t + 9t2.What is its velocity at
t=3?

a. 18

b. 54
c. 36

d. 72

Basic Engineering Correlation (Advance Mathematics and Differential Equation Reviewer)

1. Solve the equation y"+6y+9y=0subject to the conditions y(0) =-4 andy (0) = 5.

a. y = (11x-4) e-3x

b. y = (-7x-4) e-3x

c. y = (-7x-4) e3x

d. y = (-11x-4) e3x

2. Solve the homogenous equation (x2+y2) dx+2xydy=0

a. x2(x2+3y2) = c

b. x(x2+2y2)=c

c. x2(x2+2y2)=c

d. x(x2<+3y2) =c

3. The expression equivalent to ∫01+I 6z2 dz is equivalent to

a. -4+6i

b. -z+Zi

c. -3+3i

d. 4+4i

4. What can be concluded about the function that the graph below depicts?
a. The following shows an even function.

b. The graph shows symmetry with respect to x=0.

c. The graph is not a periodic function.

d. The following shows an odd function.

5. If A = 25 eπ/42 i and B = CiS π\4 then A + B is_____"

a. 39.68∠125.62o

b. 40∠75o

c. 53.26+ 32.11i

d. 32.26+23.11i

6. Which of the following power series is a solution to the differential equation y" + y' = 0 ?

a.

b.

c.

d.
7. The differential equation dv = (y2 - 3vy)dy is said to be

a. linear in y

b. non linear in V

c. linear in V

d. non linear in x

8. The laplace transform of t is

a. 1/s-1

b. 1/s

c. 2/s2

d. 1/s2

9. Determine the value of the Legendre's polynomial function P2(2).

a. P2 (2) = 2.5

b. P2(2) = 5.5

c. P2 (2) = 4

d. P2(2)=1

10. The rate at which a solid substance dissolves varies directly has the amount of undissolved solid present in the
solvent and as the difference between the saturation concentration of the substance and the instantaneous
concentration of the solution Five grams of A are placed in solvent B .the solution when saturated will hold ten
grams of A. If 2 grams of A dossolved in 1 hr, how many grams of A will be in solution in 2 hrs?

a. 7 g

b. 5 g

c. 4 g

d. 3 g
11. Find the differential equation whose general solution is y = C 1x + C2 ex.

a. (x + 1)y" - xy' + y = 0

b. (x - 1)y" + xy' + y = 0

c. (x + 1)y" + xy' +y = 0

d. (x - 1)y" - xy' + y = 0

12. A cylindrical tank is 12ft. In diameter and 8=9 ft high. Water flows into the tank at the rate of /10 cuft/sec. It
has a hole radius 1/2 inch at the bottom. The time the tank will be full if initially it is empty is

a. 76 min

b. 65 min

c. 56 min

d. 50 min

13. The indicial equation of the Bessel's equation x2 y" + xy' + (x2 - 9) y =0 is

a. r2 - 9 = 0

b. r2 + 3 = 0

c. r2 + r - 9 = 0

d. r2 + r - 3 = 0

14. The solution to the equation x2y'+xy'+x2y=0 if x=0.5 is approximately equal to

a. 0.7652

b. 0.5118

c. 0.9385

d. 0.5

15. The different equation y" + 3y' - 4y =2x is

a. first order linear different equation


b. second order non homogenous linear different equation

c. second order homogenous linear different equation

d. second order homogenous linear different equation

16. The series   is equivalent to the function

a. f(x) = e3x 

b. f(x) = 1/1-3x

c. f(x) = cos 3x

d. f(x) = sin 3x

17. The expression (3+2i)6 is equivalent to

a. -2035- 828i

b. -352+ 936i

c. 729+ 64i

d. 2187-128i

18. What is the order of the differential equation (4 + y")1/3 = e2x

a. three

b. one-third

c. one

d. two

19. The indicial equation of ODE 2xy"+(l+ x)y'-2y=o is

a. 2r2 - r =0

b. r2 -2 +l=O

c. r2-2r =0
d. r2-r =0

20. The population of a certain municipality increases at a rate to the square root of the population. If the present
population is 90,000, how long will it take for the population to reach 160,000?

a. 210 years

b. 150 years

c. 200 years

d. 180 years

21. Find the equation of the curve at every point at which the tangent line has a slope of 2x.

a. y = x2 + C

b. y = -x2 + C

c. x = -y2 + C

d. x = y2 + C

22. The order of the different equation

a. 2

b. 4

c. 3

d. 6

23. A water container whose circular cross section is 6 ft in diameter and whose height is 8 ft. is filled with water. It
has a hole at the bottom of radius 1 inch. The time it will take if the tank rests on support so that its 8 ft height is in
a horizontal direction and the hole in its bottom is

a. 25.46 min

b. 29.4 min

c. 28.95 min
d. 24.95 min

24. Determine the values of the constants r in the indicial equations of the given ordinary differential equation (2x2
— 24"-2,942y = 0 when Frobenius' method is applied.

a. ri 0,r2 = 2

b. r, = r2 = 2

c. = 1,r2= 2

d. = 0,r2 =1

25. Find a power series for the function

a. x-x3+x5 - +...

b. 1+x2+x4+...

c. 1- x2+x4+...

d. x+x3+x5+...

26. The order of the differential equation is

a. 3

b. 4

c. 1

d. 2

27. The expression ∫0πi cos z dz is equivalent to

a. 13.098

b. 23,097i

c. 13.097i
d. 11.55i

28. Solve (cos x cos y - cot x)dx - sin x sin y dy = 0

a. sin x cos y = -ln(C cos)

b. sin x cos y = ln (C sin x)

c. sin x cos y = -ln (C sin x)

d. sin x cos y = ln (C cos x)

29. The ganeral solution of the ordinary different equation with c = constant is

a. - In(1 - 2 y) = x 22 + c

b. In(1 - 2y) = x2 + c

c. - 1 In(1 - 2 y) = x22 + c

d. 2 y = 1 + ce-x2

30. The differential equation given is correctly described by which one of the following choices: d 2y/dx2 + bxy dy/dx
= f(x)

a. non-linear, second order, non homogenous

b. non linear, second order, homogenous

c. linear,second order homogenous

d. linear. Second order, non homogenous

31. Which of the following is true about the Fourier coefficients of f(x)= x if -π ≤ x ≤ π the value of f(π/2) is

a.

b.
c.

d.

32. Sugar decomposes in water at a rate proportional to the amount still unchanged. If there were 50 kg of sugar
present initially and at the end of 5 hours this is reduced to 20 kg, how long will it take until 90% os the sugar is
decomposed.

a. 12.56 hr

b. 15.72 hr

c. 16.41hr

d. 14.12 hr

33. In the higher-order differential equation (4 — x2 )y'''-4y1+y = 0 , x = —2 is a/an point.

a. focal

b. ordinary

c. regular

d. singular

34. Evaluate cos(3 + 5i)

a. -.99 + 0.28i

b. 0.53-3.59i

c. -73.47 -10.47i

d. -3.72- 0.51i

35. A new water pump has a capacity of 60 cu m/day. If its capacity goes down by 15% every year, in how many
years will the capacity go down to 20 cu m/day?

a. 4.72 yrs.
b. 7.32 yrs.

c. 8.6 yrs

d. 3.72 yrs.

36. Which of the following is the solution to the Bessel's equation x 2 y" + xy' + (x2 - y2) y=0

a.

b.

c.

d.

37. A certain quantity increases at a rate proportional to q itself. If q = 25 when t = 0 and q = 75 when t =2, find q
when t = 6.

a. 675

b. 576

c. 756

d. 657

38. Calculate the time in hrs, that it will take to reach the fatal conc. Of 40% methane in a kitchen measuring 15 ft x
12.5 ft x8 ft for a leaking stove. The rate of leak is 15 cuft of 100% methane/hr. Assume no fresh air is coming in.
The gas rate is measured at the rate conditions prevailing in the kitchen.

a. 40 hrs.
b. 50 hrs.

c. 30 hrs.

d. 45 hrs.

39. Determine the Fourier coefficient a() of the function f (x) = 3x2 + 4, —1 < x <1.

a. ao = 5

b. ao = 1

c. a = 0

d. as=10

40. The differential equation (x2 +4xy+y2)dx-xydy=0 is

a. variable separable

b. linear differential equation

c. exact

d. homogenous

41. The differential equation  can be classified as

a. exact

b. variable separable

c. linear but not homogenous

d. linear and homogenous

42. A spherical tank whose inner diameter is 2 meters is filled with water (density 1 g/cc). If a tank has a hole 1 cm
in diameter at the bottom, the time the tank will be totally empty is

a. 3.61 hrs.

b. 2.41 hrs.
c. 4.21 hrs.

d. 6.31 hrs.

43. The simplified form of (3 + 2i) is

a. 2,034-1781i

b. -2,034+1781i

c. -2,035-828i

d. 2,035+828i

44. The radius of conversence of the power series (not sure yet)

a.

b.

c.

d.

45. Which of the following is a differential equation of the first order of degree one?

a.

46. Find the differential equations og the family of lines through the origin.

a. xdy - ydx = 0

b. ydx + xdy = 0

c. xdx + ydy = 0
d. ydx - xdy = 0

47. Solve the equation

a.

b.

c.

d.

48. Solve the different equation

a. y=(2x3 + 11)2)

b. y=(2x3 - 5)

c. y=(x3 -5)2)

d. y=(x3 +11)2

49. Determine the general solution of xdy + ydx = 0

a. ln (xy) = c

b. ln x + ln y = c

c. xy = c

d. x + y = c

50. Find the equation of the orthogonal trajectories of the system of parabolas y 2=2x+C.

a. y = C ex

b. y = C e-x
c. y = C e-2x

d. y = C e2x

51. The principal 4th root of 5 + 12i

a. 1.62 + 0.39i

b. 1.49 + 1.86i

c. 0.73 + 1.75i

d. 1.82 + 0.55i

52. Evaluate 143 - 41).

a. 1.28+ j0.98

b. 1.76+ j0.54

c. 2.23+ j0.21

d. 1.61- 0.931

53. Solve

a. y= -x5+cx6

b. y=x5+cx6

c. y=-x6+cx5

d. y=x6+cx5

54. What is the differential equation of a family of parabolas having their vertices at the origin and their vertices on
the x-axis?

a. xdy + ydx = 0

b. 2ydx - xdy = 0

c. 2xdy - ydx = 0

d. dy/dx - x = 0
55. When a simple electric circuit, containing no condensers but having inductance and resistance, has the
electromotive force removed, the rate of decrease of current is proportional to the current. The current is i
amperes t seconds after the cutoff, and i = 40 when t = 0. If the current dies down to 15 amperes in 0.01 sec, fid i
after 0.1 sec.

a. 0.003amp

b. 0.001amp

c. 0.004amp

d. 0.002amp

56. Solve the differential equation : x(y - x = 1,determine y when x = 2.

a. 1.55

b. 1.63

c. 1.48

d. 1.8

57. How can the differential equation a d2x/dt2 + B(t) dx/dt + c = D(t) best be described?

a. linear, homogenous and first order

b. second order and non homogenous

c. homogenous and first order

d. linear, second order and non homogenous

58. Evaluate sin ( 3 + 4i )

a. 0.14 -0.75i

b. 3.85 - 27.02i

c. -0.96 + 4i

d. -0.09 + 0.75i
59. A body weighing 1960 N is pulled by a constant force of 492 N along a horizontal plane where in the coefficient
of friction between the body and the plane id 0.20. Determine the velocity after 20 seconds.

a. 13.1 m/s

b. 10.57 m/s

c. 8.25 m/s

d. 9.06 m/s

60. A tank and its contents weigh 100 lbs. The average heat capacity of the system is 0.5 Btu/ lb.F. The liquid in the
tank is heated by an immersion heater which delivers 100 Btu/min. Heat is lost from the system at a rate
proportional to the difference between the temperature of the system (assumed uniform throughtout at any
instant) and the temperature of the surrounding air, the proportionality constant being 2 Btu/min oF. If the air
temperature remains constant at 70oF and if the initial temperature of the tank and its contents is 55 oF, the
temperature of the tank as a function of the is

a. T=120+65et/25

b. T=12-6.5e-t/25

c. T=120-65e-t/25

d. T=-120+65e-t/25

61. Which of the following is a solution of the wave equation

a. u=ex cos t

b. u =(x + at)6

c. u = ln(ax-t)

d. u = sin(kx)sin(at)

62. A low radioactive material is used in biochemical process to induce biological mutation. The isotope is made in
the experimental reactor of the Philippine Atomic Energy Commssion, now Philippine Nuclear Research Institute,
and ship to the chemical plant. It has a half life of 8.06 days. The plant receive the shipment of the radioactive
material which on arrival contain 1 gram of the radioactive material. The plant uses the material at the rate of 0.1
gram per week. The time it will take for the radioactivity to last is

a. 4.74 weeks
b. 3.24 weeks

c. 5.4 weeks

d. 4.34 weeks

63. Solve the differential equation dy - xdx = 0, if the curve passes through (1, 0).

a. 3x2 + 2y - 3 = 0

b. 2y + x2 - 1 = 0

c. 2x2 + 2y - 2 = 0

d. x2 - 2y -1 = 0

64. A 10-ohm resistor and a 5-henry inductor are connected in series with to a 50-volt source at time t = 0. Express
the current I as a function of time.

a. i = 5(1 - e)2t

b. i = 5(e2t - 1)

c. i = 5(1 - e-2t)

d. i = 5(1 - e2t)

65. Evaluate cosh(5 + 6i)

a. 201.72 +74.21i

b. 57.22-193.43i

c. 71.25-2073i

d. 74.20 - 0.28i

66. A 50 lb iron ball is heated to 200oF and then plunged immediately into a vessel containing 100b lbs of water
whose temperature is 40oF. The specific heat of iron is 0.11 Btu/lb oF. The common temperature, approached by
the iron and water as time approaches infinity is

a. 68.5oF

b. 58.4oF
c. 48.34oF

d. 38.43oF

67. The rate f decay of radioactivity elements is usually assumed to be proportional to the number of atoms that
have not decayed, where &#955 is the proportionality consatnt. If at time t=0 there are X o atoms of a given
elements, the expression for the number of atoms, X, that have not decayed (as a function of time,t,&#955, and X o)
is

a. Xo/(1+λt)

b. Xo(1-λt)

c. Xoe-λt

d. Xoe(1-e-λt)

68. Which of the following is a term of the power series representation solution of the higher order differential
equation 3 y" —2 x y = 0

a. a1

b. 4

c. 4a2

d. 1

69. The solution to the non homogeneous partial differential equation

a. u(x,y)=f(y)e2x-4x

b. u(x,y)=f(x)e-2y+4y

c. u(x,y)=f(x)e-2y+2y-1

d. u(x,y)=f(y)e-2x-2x-1

70. Find the general solution of y' = ysec x.

a. y = C sec x tan x
b. y = C (sec2 x - tan y)

c. y = C (sec x - tan x)

d. y = C (sec x + tan x)

71. Determine the value of c such that the function u(x,t) = e -256 sin 2x will be a solution of the heat equation

given by 

a. 1

b. 4

c. 8

d. 2

72. The expression (5+2i)7 is equivalent to

a. -15939+ 1846C1

b. -703919-68880i

c. -116615+60422i

d. 78125+128i

73. Find the principal 5th root of 5+121.

a. 1.64 +1.38i

b. 1.38+1.641

c. 1.67+0.13i

d. 1.62+0.391

74. Evaluatecos(2+3i).

a. -2,034+17811

b. 2,035+828i
c. 2,034-1781i

d. -4.19-9.11i

75. A body whose temperature is 180o is immersed in a liquid which is kept at a constant temperature of 60 o. In 10
minutes the temperature of the immersed body decreased to 120 o. How long will it take for the body's temperature
to decrease to 90o?

a. 15 min.

b. 20 min.

c. 25 min.

d. 18 min.

76. the equation y2 = cx is the general solution of

a. y' = x/2y

b. y' = 2y/x

c. y' = y/2x

d. y' = 2x/y

77. Find the radius of the convergence of the series

a. |x| < 2

b. |x| < ½

c. |x| < 8

d. |x| < 1/8

78. Radium decomposes at a rate proportional to the amount at any instant. In 100 years, 100 mg of radium
decomposes to 96 mg. How many mg will be left after 100 years?

a. 88.6
b. 90.72

c. 92.16

d. 95.32

79. A certain subxtance increases at a rate proportional to the square of the instantaneous amount. After 5 days
the amount is doubled. Determine the time before the amount is tripled.

a. 40/3

b. 45/3

c. 20/3

d. 25/3

80. Evaluate sinh(6 + 5i)

a. 57.22 –193.43i

b. 201.71+ 74.201

c. –20.74 + 71.25i

d. –0.27 – 0.96i

81. Which of the following is true about the Fourier coefficients of 

a. ao=7

b. ao= 0

c. ao=10

d. ao=5

82. The solution to the homogeneous partial differential equation


a. u(x,y)=A(y)cos 3x+B(y)sin 3x

b. u(x,y)=A(y)e3x +B(y)xe3x

c. u(x,y)=A(y)e3x +B(y)e-3x

d. u(x,y)=A(y) +B(y)e-9x

83. Solve xy'(2y -1) = y(1-x)

a. ln (xy) = x + 2y + C

b. ln (xy) = 2y - x + C

c. ln (xy) = x - 2y + C

d. ln (xy) = 2 (x - y) + C

84. A tank initialy contains 400 liters of water. Salt solution, containing 1/8 kg of salt per liter of solution flows into
the tank at the rate of 8 li/min and the solution, kept well-stirred, flows out of the tank at the rate of 4 li/min. Find
the amount of salt in the tank after 100 minutes.

a. 80 kg

b. 85 kg

c. 75 kg

d. 70 kg

85. A mothball loses mass by evaporation at rate that is proportional to the surface area. If half tha mass is lost in
100 days, how long will it take the radius to decreases to half its initial value?

a. 255 days

b. 275 days

c. 243 days

d. 234 days

86. The laplace transform of et is

a. 1/(s-1)2
b. 1/(s+1)

c. 1/(s-1)

d. 1/s

87. Evaluate cosh ( 3 + 5i)

a. 2.86 + 9.61i

b. 1.61 + 0.93i

c. 2.08 + 1.79i

d. 2.08 + 0.93 i

88. If dy = x2dx, what is the equation of y in terms of x if the curve passes through (1,1)?

a. x3 - 3y + 2 = 0

b. x3 + 3y + 2 = 0

c. x2 - 3y + 3 = 0

d. 2y + x3 + 2 = 0

89. Evaluateln(5 +j3).

a. 1.28+ j0.98

b. 2.54+ j0.866

c. 2.23+ j0.21

d. 1.76+ j0.54

90. Which of the following power series is a solution to the differential equation

a.
b.

c.

d.

91. Solve the equation

a. y =cIe5x+ c2e3x

b. y =cIe-5x+ c2e-3x

c. y =(cIx+ c2)e-5x

d. y =(cIx+ c2)e3x

Basic Engineering Correlation (Chemistry Reviewer)

1. Uranium-235 and uranium-238 have the same number of which of the following?

a. Protons and electrons

b. neutrons

c. protons

d. electrons

2. What is the valence (oxidation state) of carbon in sodium carbonate (Na 2CO3)?

a. -4
b. 4

c. 2

d. -2

3. Water and SO3 combine to sulfuric acid (H2SO4) according to the following reaction. How many grams of water
must be added to 100 g of 20% oleum (20% SO3 and 80% H2SO4by weight) to produce a 95% solution ( byweight) of
sulfuric acid?

a. 3.3 g

b. 14 g

c. 5.0 g

d. 7.5 g

4. During a laboratory experiment at 1.0 atm and 25 oC, a student observed that oxygen gas was produced by de-
composition of 15 g of sodium chlorate. What was the volume of oxygen?

a. 1.27 L

b. 6.54 L

c. 5.17 L

d. 3.85 L

5. Which of the following does a catalyst change?

a. the activation energy of a reaction

b. the equilibrium constant of areaction

c. the concentration of product at equilibrium

d. the heat of reaction of a reaction

6. What is an isomer?

a. a substance containing a hydroxyl ion

b. a single atom
c. different arrangement of the same atoms

d. a basic building block for large chemical chains

7. The reaction shown occurs in a gaseous phase. Once equilibrium has been achieved in a particular reaction
vessel, additional HI gas is injected directly into the reaction vessel. Compared to the initial conditions, which of
the following statemnets is correct after the new equilibrium has been achieved?

a. The amount of H2 will have decreased.

b. The partial pressure of H2 will have decreased.

c. The amount of I2 will have increased.

d. The partila pressure of HI will have decreased.

8. Which of the following compounds would be ionic, considering the electronegativities of the elements?

a. I2

b. NO

c. CO

d. KCI

9. An unknown gas with a temperature of 25oC and a pressure of 740 mm Hg is collected in a sampling bag. The
volume and mass of the gas are 24.0 L and 34.9 g, respectively. Which chemical formula could represent the gas?

a. N2

b. H2S

c. HCI

d. Ar

10. What is the percentage (by mass) of htdrogen in glucose (C 6H12O6)?

a. 0.067

b. 0.093

c. 0.17
d. 0.4

11. 2.00 g of a substance dissolved in 250 g of water produces a boiling point elevation of 0.065 oC. What is the
molecular weight of the substance?

a. 63

b. 92

c. 16

d. 8

12. A current of 0.075 A passes through a solution of silver nitrate for 10 munites. How much silver is deposited?

a. 0.040g

b. 0.035 g

c. 0.030 g

d. 0.050 g

13. How many grams of copper will be deposited at an electrode if a current of 1.5 A is supplied for 2 hours to a
CuSO4?

a. 7.1 g

b. 3.6 g

c. 48 g

d. 2.4 g

14. What is the order of reaction with respect to reactant E and the overall order of the reaction described by the
following rate law?

a. second order with respect to E; second order overall

b. first order with respect to E; second order overall

c. second order with respect to E; fourth order overall

d. first order with respect to E; fourth order overall


15. What is the term for a quantity of a susbstance to which a chemical formula can be assigned and whose mass is
equal to its formula weight?

a. a mole

b. an equivalent

c. a molecule

d. a one-normal solution

16. The pH of a 0.001 M HCI solution is

a. 5

b. 3

c. 7

d. 1

17. The half-life of radioactive carbon is approximately 5700 years. If a sample is found to have 7000 atoms after
6000 years, how many atoms were presents initially?

a. 13800 atoms

b. 14500 atoms

c. 14300 atoms

d. 14100 atoms

18. Given the following reversible chemical reaction, assume all reactants and products are ideal gases.

a. The amount of ammonia (NH3) would halve.

b. There would be no change in the amount of ammonia (NH 3) present.

c. More ammonia (NH3) would be generated.

d. The amount of ammonia (NH3) would double.

19. Which of the following statements concerning reversible reactions is false?


a. Temperature affects the direction of the reaction.

b. Concentration have no effect on the direction of the reaction

c. Concentration remain constant once equilibrium is reached.

d. Both reactants and products are always present.

20. Which of the following reactions are not balanced?

a. IV only

b. I only

c. II and III

d. I and III

21. 2.00 g of a substance dissolved in 250 g of water produces a boiling point elevation of 0.065 oC. What is the
molecular weight of the substance?

a. 92

b. 16

c. 8

d. 63

22. Oxygen reacts stoichimetrically with methane to form 14 g of carbon monoxide. How many moles of methane
are consumed?

a. 1 mol

b. 0.5 mol

c. 2 mol

d. 1.5 mol

23. Which o fthe following chemical formulas is incorrect?

a. Na2CO3

b. KOH
c. Ca(OH)2

d. CaCI

24. What are the chemical formulas for the following compounds: aluminum nitrate, magnesium hydroxide,
calcium oxide, and cupric carbonate?

a. AINO3,Mg(OH),Ca2O3,CuCO3

b. AI(NO3)3,Mg(OH)2,CaO,CuCO3

c. AL2NO3,Mg(HO),CaO2,CuCO3

d. AINO3Mg(HO)2,CaO,Cu(CO3)2

25. Nitroglycerin is made by combining glycerol, nitric acid, and sulfuric acid. What are the minimum coefficients
needed to balance the equation of this reactions?

a. 4,2,1,1,3,1

b. 1,3,1,1,3,1

c. 1,3,3,1,3,2

d. 2,6,2,2,6,2

26. Which of the following occurs when table salt (NaCI) is added to continuously heated boiling water?

a. The water boils even more agitatedly.

b. The temperature of the water decreases but boiling continues uninterrupted.

c. The water continues to boil.

d. The water momentarily stops boiling.

27. The final temperature of the hydrogen and chlorine described in Prob. 8 is 30 oC. What is the final pressure in
the reaction vessel?

a. 80 kPa

b. 320 kPa

c. 240 kPa
d. 160 kPa

28. A wastewater treatment plant uses chlorine gas as a reactant. A tank is filled with 800 m 3 of 20oC water, and
chlorine is added at a dosage of 125 g per cubic meter of water. (Assume all of the chlorine dissolves and none
initially reacts chemically.) If the atmospheric pressure is 1.0 atm, what is the theoretical partial pressure of the
chlorine gas at the tank surface immediately after the gas is added?

a. 2.3 x 10-4 atm

b. 0.11 atm

c. 3.1 x 10-5 atm

d. 0.039 atm

29. What family of compounds is produced from the reaction between an alcohol and a carboxylic acid?

a. ether

b. amine

c. ester

d. ketone

30. Which of the following statements pertaining to acids and bases is incorrect?

a. Acids conduct electricity in aqueous solutions.

b. Bases have a pH between 7 and 14.

c. Bases have a sour taste.

d. Acids turn blue litmus paper red.

31. As the pressure of a gas increases, the solubility of that gas in a liquid

a. always increases.

b. is not changed.

c. always decreases.

d. cannot be determined.
32. What is a distinguishing characteristic of the halogens?

a. They are phosphorescent.

b. Next to the noble gases, they are the most chemicallyinactive group.

c. They readily accept an electron from another atom to form compounds.

d. They have a high electrical conductivity.

33. Enthalpy of formation is most closely defined as the

a. potential energy of a substance.

b. energy absorbed during creation of 1 grammole of a compound from pure elements.

c. energy absorbed or released during a chemical reaction.

d. sum of the enthalpy of reactions.

34. Assuming all of the energy goes into the reaction, what electrical power is required to produce oxygen gas at a
rate of 50 mg/s?

a. 9.2 kW

b. 0.89 kW%

c. 3.1 kW

d. 1.5 kW

35. What is the oxidation number for chromium (Cr) in the compound BaCro?
a. 2

b. 4

c. 1

d. 6

36. A transportation company specializes in the shipment of pressurized gaseous materials. An order is received for
100 L of a particular gas at STP (0oC and 1 atm). What minimum volume tank is necessary to transport the gas at
25oC and a maximum pressure of 8 atm?

a. 14 L

b. 12 L

c. 16 L

d. 10 L

37. In an experient, a compound was determined to contain 68.94% oxygen and 31.06% of an unknown element by
weight. The molecular weight of this compound is 69.7 g/mol. What is this compound?

a. SiO4

b. NO2

c. F2O2

d. B2O3

38. 6 g of a substance are dissolved in 1000 g of water. The solution freezes at -0.16 oC. What is the molecular
weight of the substance?

a. 70 g/ mol

b. 60 g/mol

c. 75 g/mol

d. 100 g/mol

39. A gaseous mixture consists of 2 kg of oxygen, 5 kg of nitrogen, and 3 kg of xenon. What is the mole fraction of
the oxygen gas?
a. 0.24

b. 0.17

c. 0.11

d. 0.13

40. The diameter of a spherical mothball is observed to halve in 200 days.approximately how long will it take for its
remaining volume to become half of its volume at 200 days?

a. 67 days

b. 160 days

c. 130 days

d. 200 days

41. For a given isotope of an element, the atomic number plus the atomic weight is 148, and their difference is 58.
how many protons does an atom of the isotope contain?

a. 45

b. 90

c. 148

d. 58

42. 10 g of solid PCI5 is heated in a 0.5 m3 container to 150oC, producing gaseous PCI3 and CI2 gas according to the
following decomposition reaction: The molecule weights of the compound are What is the increase in pressure in
the container when 50% (By weight) of the PCI5 is decomposed?

a. 0.250 kPa

b. 18 kPa

c. 0.120 kPa

d. 0.350 kPa

43. An alkyl radical is best defined as

a. an electron that is shared in a covalent bond.


b. any functional group that substitutes for a hydrogen atom in an alkane.

c. the remaining portion of an alkane after it loses a hydrogen atom.

d. cancer-causing molecules found in foods

44. It is known that ozone (O3) will decompose into oxygen (O2) at a temperature of 100o. One mole of ozone is
sealed in a container at STP (0oC and 1 atm). What will be pressure of the container once it is heated to 100 oC?

a. 37 kPa

b. 1.4 kPa

c. 2.1 kPa

d. 210 kPa

45. How much water must be added to 100 mL of a 0.75 molar solution of KCI to make a 0.04 molar solution?

a. 1.88 L

b. 0.188 L

c. 1.78 L

d. 1.98 L

46. While moving from left to right across the second row of the periodic table (i.e., from Li to Ne), the atomic radii
tend to

a. first increase, then decrease.

b. uniformly increase.

c. remain the same.

d. uniformly decrease.

47. A solution is adjusted from pH 8 to pH 9. The relative concentraation of the hydrogen [H +] ion has changed by a
factor of what?

a. 1100

b. 5
c. 110

d. 10

48. The solubility constant of stronyium sulfate, SrSO 4, is 2.8 x 10-7. How many grams of SrSO4 must be dissolved in
water to produce 1 L saturated solution?

a. 0.1 g

b. 2 g

c. 0.00005 g

d. 0.0005 g

49. Which of the following elements has the largest first ionazation energy?

a. CI (chlorine)

b. Ar (argon)

c. H (hydrogen)

d. Kr (krypton)

50. How many milliters of 1 M NaOH solution will 25 mL of 2 H 2SO4neutralize?

a. 50 mL

b. 100 mL

c. 75 mL

d. 25 mL

51. What is the molarity of a solution obtained by dis-solving 25 g of NaCI in enough water to produce 4 L of
solution?

a. 6.25

b. 0.365

c. 0.428

d. 0.107
52. What is the half-life of a substance that decays to 25% of its original amount in six days?

a. 3 days

b. 0.08 days

c. 8 days

d. 12 days

53. Two moles of sodium react with 2 moles of water to produce which of the following?

a. 1 mole of sodium hydroxide and 1 mole of hydrogen

b. 2 moles of sodium hydroxide and 1 mole of hydrogen

c. 1 mole of sodium hydroxide and 2 moles of hydrogen

d. 2 moles of sodium hydroxide and 2 mole of hydrogen

54. When a deliquescent substance is exposed to air, it

a. oxidizes.

b. becomes moist

c. crystallizes.

d. loses water of hydration.

55. Which of the following elements does not exists as a diatomic molecule under normal (ambient) conditions?

a. chlorine

b. oxygen

c. iodine

d. sulfur

56. A given sample of radioactive material has 80% of the original substance remaining after 10 years. How much
will remain after 90 additional years?
a. 0.001

b. 0.017

c. 0.11

d. 0.13

57. The decay of U-238 to Pb-206 can be used to estimate the age of inorganic matter. The half-life of U-238 is 4.5
x 109 years. In a particular rock sample, the ratio of the numbers of Pb-206 to U-238 atoms is 0.66. Assume all of
the Pb-206 present is due to the decay of U-238. What is the age of the rock?

a. 3.3x 109 yr

b. 1.4 x 109 yr

c. 7.0 x 109 yr

d. 9.3 x 109yr

58. How much energy is needed to convert ozone to oxygen?

a. 43 kcal/mol

b. 0 kcal/ mol

c. 68 kcal/mol

d. 140 kcal/mol

59. The group of metals that includes lithium, sodium,potassium, rubidium, and cesium forms a closely related
family known as the

a. rare earth group.

b. halogens.

c. alkali metals.

d. alkaline earth metals.

60. Hydrogen and chlorine gas combine in a 35 m 3 reaction vessel to produce hydrogen chloride. The masses of
hydrogen and chlorine are 4.5 kg and 160 kg, respectively. How much hydrogen chloride gas is produced?

a. 41 kg
b. 21 kg

c. 82 kg

d. 160 kg

61. How many liters of 2M solution (i.e., a molarity of 2) can be produced from 184 g of enthyl alcohol
(CH3CH2OH)?

a. 1.5 L

b. 5.0 L

c. 2.0 L

d. 2.5 L

62. If the current, I, is 100 A, at what rate is oxygen produced?

a. 18.7 mg/s

b. 8.29 mg/s

c. 16.7 mg/s

d. 9.34 mg/s

63. What mass of lead nitrate, Pb(NO3)2, must be dis-solved in 1 L of water to produce a solution that contains 20
mg of lead ions? Assume 100% ionazation.

a. 43 mg

b. 52 mg
c. 32 mg

d. 26 mg

64. A compound in gas form Has a mass of 0.377 g and occupies 191.6 mL at standard conditions (0 oC and 760 mm
Hg). What is the formula of the compound?

a. C3H8

b. CH4

c. C5H12

d. C2H6

65. A gas mixture of N2(g) and CO2(g) contained in a volume of 10.0 L has a total pressure of 0.750 atm at a
temperature of 273K. The mixture is known to contain 3.00 g N2(g). What is the partial pressure of CO2(g) in the
mixture?

a. 0.120 atm

b. 0.630 atm

c. 0.510 atm

d. 0.240 atm

66. Rank the following gas according to increasing effusion rates relative to O2 (reference).

a. F2< CO24

b. F<2< CH<4< CO<2

c. CH42<CO2

d. CO22<2

67. What is the vapor pressure of 1000.0 g of a water solution at 250C that contains 124.0 g of the nonvolatile
solute ethylene glycol, C2H6O2? The vapor pressure of pure water at this temperature is 23.76 torr. Assume an
ideal solution

a. 23.7 torr

b. 24.6 torr
c. 22.8 torr

d. 0.94 torr

68. "::Add_Chem_004:: Consider a solution of water and a nonvolatile solute at some temperature. What
combination of conditions would be sure to increase the vapor pressure of the solution? _____"

a. Raise the temperature and add more water

b. Lower the temperature and add more solute

c. Raise the temperature and add more solute

d. Lower the temperature and add more water

69. What is the mass of 0.01 gram-moles of Na2SO4?

a. 1.42 g

b. 1.19 g

c. 0.71 g

d. 2.38 g

70. An ideal gas occupies a volume of 4L and has a pressure of 283.71kPa (1atm=101.325kPa). Under 22.50C, what
most likely is the identity of the gas if 0.01293 Kg of gas is used.

a. O2

b. Cl2

c. F2

d. N2

71. The reaction shown proceeds in a gaseous state. At equilibrium, the concentration of the components X,Y, and
Z are measured to be 5.73 x 10-2 mol/L, 2.67 x 10-2 mol/L and 4.59 x 10-2 mol/L, respectively. What is the
equilibrium constant for this reaction?

a. 9.8 x 10-4mol/L

b. 1.7 x 10-2mol/L

c. 3.7 x 10-1 mol/L


d. 2.1 x 10-2mol/L

72. If 1.5 L of an ideal gas at 250C is heated, the new volume increases 2.5 times the original volume. The pressure
and amount of substance are held constant. What is the new temperature of gas in 0F

a. 882 0F

b. 8800F

c. 820 0F

d. 8280F

73. "::Add_Chem_013::Following are three states for fluorine:1s22s12p6 1s22s22p5 1s22s22p42d1 They are,
respectively: _____"

a. ground, excited, impossible

b. ground, impossible, excited

c. excited, impossible, ground

d. excited, ground, impossible

74. At what temperature in 0C will O2 has under a pressure of 2.4 atm? (ρ of O2= 1.43 g/L)

a. 645.7510C

b. 3810C

c. 315.570C

d. 318.160C

75. Macro Vee collected hydrogen gas using water displacement method. He measured the temperature of water
using a thermometer and found out that it is 230C with the correponding pressure of 21.1 mmHg. Calculate the
pressure of hydrogen gas under standard atmospheric pressure.

a. 95.81 KPa

b. 98.51 Kpa

c. 98.15 Kpa

d. 95.18 Kpa
76. A sample of an unknown compound is found to be 49.3% carbons, 9.6% hydrogen, 19.2% nitrogen, and 21.9%
oxygen by weight. What is its molecular formula?

a. C3H7NO

b. C3H7NO

c. C4H<><>

d. C4H4NO

77. Balance the following reaction.

a. HBrO3 + 5HBr 3H2O + 3bR2

b. 3HBrO3 + HBr 2H2O + 2Br2

c. 2HBrO3 + 4HBr 3H2O + 3Br2

d. HBrO3 + 4HBr 3H2O + Br2

78. Which of the following is the correct electron configuration of Pb?

a. [Xe]6s24f146s2

b. [Xe]6s25d104f146p2

c. [Xe]6s25d104f145d106p2

d. [Xe]5d104f146p2

79. The mole (mol) is the amount of a substance that contains as many elementary entities as there are atoms in
exactly

a. 12.00 grams of C.

b. average atomic mass of isotopes of C.

c. 12.01 grams/mol of C.

d. 12.00 grams of 12C.

80. At STP the volume of 1.5 mole N2as compared to 1.0 mole O2 is
a. higher to about three fourths

b. the same, 22.4L

c. different by about 11.2 L

d. differ by a factor of 1.25

81. "::Add_Chem_008:: How many grams of glucose, C6H12O6, are necessary to prepare 656 mL of a solution with
a concentration that is 0.550 molar? _____"

a. 0.00200 g

b. 151 g

c. 64.9 g

d. 214 g

82. n the following reaction, which elements are the reducing and exidizing agents?

a. Mg is the reducing agents; O2 is the oxidizing agent.

b. MgO is the reducing agent; Mg is the oxidizing agent.

c. Mg is the reducing agent;MgO is the oxidizing agent.

d. O2 is the reducing agent; Mg is the oxidizing agent.

83. By decreasing the pressure of an ideal gas at constant temperature and amount of substance 1/3 times the
original pressure, the volume of gas will

a. expands two thirds the original

b. increases three times the original

c. multiply by a factor of 1/3

d. decreases three times the original

84. What is the gravimetric (i.e.m.,mass) percentage of oxygen in K 2CrO4?

a. 0.33

b. 0.66
c. 0.57

d. 0.42

85. There are 500 g of zinc sulfide (ZnS) in a load of zinc ore. The ZnS is roasted in excess air to form zinc oxide
(ZnO) and sulfer dioxide (S)2). How many grams of zinc can be subsequently recovered if 5% of the zinc is lost in the
roasting process?

a. 340 g

b. 380 g

c. 320 g

d. 400 g

86. What is the enthalpy of reaction at 25oC for the combustion of ethane ( C2H6)?

a. -680 kcal/mol (exothermic)

b. -340 kcal/mol ( exothermic)

c. 130 kcal/mol (endothermic)

d. 340 kcal/mol (endothermic)

Basic Engineering Correlation (Physics Reviewer)

(A Collaborative work of GaMbit, jay729, and airsWTP)

1. The system shown is in static equilibrium. Find W.

Select one:

a. 1000 N

b. 1700 N

c. 1500 N
d. 830 N

2. What is the force in member AF?

Select one:

a. 5000 N

b. 15 000 N

c. 10 000 N

d. O

3. A ball is dropped from rest at a point 12 m above the ground into a smooth, frictionless chute. The ball exist the
chute 2 m above the ground and at angle 45o from the horizontal. Air resistance is negligible. Approximately how
far will the ball travel in the horizontal direction before hitting the ground?

Select one:

a. 22 m

b. 20 m

c. 24 m

d. 12 m
4. The structure shown is formed of three separate solid aluminum cylindrical rods, each with a 1 cm diameter.
What is the -coordinate of the centroid of volume for the structure?

Select one:

a. 15.2 cm

b. 16.0 cm

c. 15.9 cm

d. 14.0 cm

5. A projectile has an initial velocity of 110 m/s and a launch angle of 20o from the horizontal. The surrounding
terrain is level, and air friction is to be disregarded. What is the maximum elevation achived by the projectile?

Select one:

a. 350 m

b. 72 m

c. 140 m

d. 620 m

6. What are R1 and R2? (insert question #11)

Select one:

a. 1250 N

b. 1250 N; R2 /

c. 1000 N; R2 /

d. R1 /

e. 3750 N

f. 4000 N
g. 2500 N; R2 /

h. 1250 N

i. R1 /

j. R1 /

k. 3750 N; R2 /

l. R1 /

7. What is the reaction at point A?

Select one:

a. 710 N

b. O

c. 500 N

d. 290 N
8. A turntable capable of angularly accelerating at 12 rad/s2 needs to be given an initial angular velocity if it is to
rotate through a net 400 radians in 6 seconds. What must its initial angular velocity be?

Select one:

a. 33 rad/s

b. 21 rad/s

c. 200 rad/s

d. 28 rad/s

9. A 550 kg mass initially at rest acted upon by a force of 50 et N. What are the acceleration, speed, and
displacement of the mass at t = 4 s?

Select one:

a. 4.96 m/s2,4.87 m/s,19.5 m

b. 4.96 m/s2,135.5 m/s,2466 m

c. 4.96 m/s2,271 m/s,3900 m

d. 4.96 m/s2,4.96 m/s,19.8 m

10. A constant force of 750 N is applied through a pulley system to lift a mass of 50 kg as shown. Neglecting the
mass and friction of the pulley system, what is the acceleration of the 50 kg mass?

Select one:

a. 20.2 m/s2

b. 16.2 m/s2

c. 8.72 m/s2

d. 5.20 m/s2

11. A child keeps a 1 kg toy airplane flying horizontally in a circle by holding onto a 1.5 m long string attached to its
wing tip. The string is always in the plane of the circular flight path. If the plane flies at 10 m/s, what is the tension
in the string?

Select one:

a. 15 N

b. 28 N
c. 7 N

d. 67 N

12. One newton is the force required to

Select one:

a. give a 1 g mass an acceleration of 1m/s2.

b. accelerate a 10 kg mass at a rate of 0.10 m/s2.

c. accelerate a 1 kg mass at a rate of 9.81 m/s2

d. accelerate a 1 kg mass at a rate of 1.00 cm/s2.

13. What is the approximate centroidal polar moment of inertia of the area?

Select one:

a. 27.3cm4

b. 25.6 cm4

c. 16.2 cm4

d. 21.4 cm4

14. A 4-A current is maintained in a simple circuit with a total resistance of 2 . How much energy is delivered in 3
seconds

Select one:

a. 3J

b. 12J

c. 6 J

d. 96J
15. In the pin-jointed truss shown, what is the force in member DE?

Select one:

a. 3500 N

b. 2500 N

c. O

d. 550 N

16. Link AB of the linkage mechanism shown in the illustration rotates with an instantaneous counterclockwise
angular velosity of 10 rad/s. What is the instantaneous angular velocity of link BC when link AB is horizontal and
link CD is vertical?

Select one:

a. 3.25 rad/s (counterclockwise)

b. 2.25 rad/s (clockwise)

c. 12.5 rad/s (clockwise)

d. 5.50 rad/s (clockwise)


17. Why does a spinning ice skater's angular velocity increase as she brings her arms in toward her body?

Select one:

a. Her angular momentum is constant

b. Her radius of gyration is reduced.

c. Her mass moment of inertia is reduced.

d. all of the above

18. A flywheel rotates at 7200 rev/min when the power is suddenly cut off. The flywheel decelerates at a constant
rate of 2.1 rad/s2 and comes to rest 6 min later. How many revolutions does the flywheel make before coming to
rest?

Select one:

a. 390 000 rev

b. 18 000 rev

c. 22 000 rev

d. 72 000 rev

19. Two 2 kg block are linked as shown. Assuming that the surfaces are frictionless, what is the velocity of block B if
block A is moving at a speed of 3 m/s?

Select one:

a. 1.30 m/s

b. 0 m/s

c. 5.20 m/s

d. 1.73 m/s

20. Find the distance between position B and C.


Select one:

a. 3.23 m

b. 10.1 m

c. 4.78 m

d. 7.78 m

21. A weekend plumber, unable to loosen a pipe fitting, slips a piece of scrap pipe (a "cheater") over his wench
handle. He then applies his full mass of 100 kg to the end of the cheater by standing on it. The distance from the
center of the fitting on the point where the weight acts is 0.80 m and the wrench handle and cheater make an
angle of 19° with the horizontal. Find the magnitude and direction of the torque he applies about the center of the
pipefitting.

Select one:

a. 740 N

b. 120 N

c. 360 N

d. 520 N

22. A 1530 kg car is towing a 300 kg trailer. The coefficient of friction between all tires and the road is 0.80. How
fast can the car and trailer travel around an unbanked curve of radius 200 m without either the car or trailer
skidding?

Select one:

a. 143 km/h

b. 75.2 km/h

c. 40.0 km/h

d. 108.1 km/h
23. A rope passes over a fixed sheave as shown. The two rope ends are parallel. A fixed load on one end of the
rope is supported by a constant force on the other end. The coefficient of friction between the rope and the
sheave is 0.30. What is the ratio of tensile forces in the two rope ends?

Select one:

a. 2.6

b. 1.6

c. 1.2

d. 1.1

24. If the sum of the forces on a particle is not equal to zero,the particle is

Select one:

a. moving with a constant velocity opposite to the direction of the resulatnt force.

b. accelerating in a direction opposite to the resultant force.

c. accelerating in the same direction as the resultant force.

d. moving with constant velocity in the direction of the resultant force.

25. What is the -coordinate of the centroid of the perimeter line?

Select one:

a. 1.66 cm

b. 1.56 cm
c. 1.75 cm

d. 1.80 cm

26. An angle bracket is subjected to the forces and couple shown. Determine the equivalent force-couple system at
point A

Select one:

a. 292 N at -5.9o ; 103 N.m

b. 333 N at 42.9o ; 53 N.m

c. 114 N at 15.3o ; 50 N.m

d. 307 N at 10.4o ; 110 N.m

27. In the figure, a very small toy race car of mass m is released from rest on the loop-the-loop track. If it is
released at a height 2R above the floor, how high is it above the floor when it leaves the track, neglecting friction?

Select one:

a. 1.33 R

b. 2.00 R

c. 1.67 R

d. 1.50 R

28. Find the acceleration of block A after the blocks are released.
Select one:

a. 2.5 m/s

b. 0 m/s

c. 1.4 m/s

d. 5.6 m/s

29. Where can a couple be moved on a rigid body to have an equivalent effect?

Select one:

a. along the perpendicular bisector joining the two original forces

b. along the line of action

c. anywhere on the rigid body

d. in a parallel plane

30. What is the reaction at point B?

Select one:

a. 20 000 N

b. 10 000 N

c. 15 000 N

d. 5000 N

31. Find the -and y-coordinates of the centroid of wire ABC


Select one:

a. 0.43 m ; 1.29 m

b. 2.71 m ; 1.49 m

c. 3.33 m ; 2.67 m

d. 0.64 m ; 2.83 m

32. For a force to do work it must be ____ the displacement

Select one:

a. shorter than

b. equal in magnitude to

c. paralllel or antiparallel to

d. perpendicular to

33. For which of the following situation is the net force acting on a particle necessarily equal to zero?

Select one:

a. The particle has constant loinear momentum.

b. The particle has constant angular momentum.

c. The particle has constant kinetic energy.

d. The particle is traveling at constant velocity around a circle.

34. A perfect sphere moves up a frictionless incline. Which of the following quantities increases?

Select one:

a. potential energy

b. total energy
c. angular velocity

d. linear momentum

35. A single force (not shown) is applied at point B in the y-direction, in line with points A and B. What should this
force bein order for the frame to be in equilibrium in that direction?.

Select one:

a. -280 N (down)

b. 120 N (down)

c. 180 N (down)

d. -250 N (down)

36. A cable passes over a stationary sheave and supports a 60 kg bucket, as shown. The coefficient of friction
between the cable abd the sheave is 0.10. The cable has a uniform mass per unit length of 0.4 kg/m. The cable is in
the shape of a catenary due to its own weight. The tension o fthe cable at the pulley is given by T = wy, where w is
the weight per unit lenght and the constant y (for this configuration) is known to be 151 m. How much more mass
can be added to the4 bucket before the cable slips over the pulley?
Select one:

a. 12.1 kg

b. 11.6 kg

c. 10.0 kg

d. 0

37. The moment of inertia about the -axis o fthe cross section shown is 245833 cm4. If the cross-sectional area is
250 cm2 and the thickness of the web and the flanges are the same, what is the moment of inertia about the
centroidal axis?

Select one:

a. 2.1 x 104 cm4

b. 1.5 x 105 cm4

c. 2.5 x 105 cm4

d. 8.0 x 104 cm4

38. Assume that the centroidal moment of inertia of area A2 with respect to the composite centroidal -axis is 73.94
cm4. The moment of inertia of area A2 with respect to the composite centroidal horizontal axis is 32.47 cm4. What
is the moment of inertia o fthe composite area with respect to its centroidal -axis?
Select one:

a. 560 cm4

b. 460 cm4

c. 480 cm4

d. 350cm4

39. Find the tension, T, that must be applied to pulley A to lift the 1200 N weight.

Select one:

a. 600 N

b. 300 N

c. 100 N

d. 400 N

40. What are the - and y-coordinates of the centroid of the area?

Select one:

a. 3.0 cm ; 4.0 cm

b. 2.4 cm ; 3.4 cm

c. 3.0 cm ; 3.6 cm

d. 3.0 cm ; 3.8 cm
41. Determine the force in member FH for the pi-connected truss shown.

Select one:

a. 4130 N (tension)

b. 0

c. 2320 N (compression)

d. 3840 N (tension)

42. What is the period of a pendulum that passes the center point 20 times a minute.

Select one:

a. 0.2 s

b. 3 s

c. 6 s

d. 0.3 s

43. A 2kg block rests on 34o incline. If the coefficient of static friction is 0.2, how much additional force, F, must be
applied to keep the block from sliding down the incline?

Select one:

a. 14 N

b. 9.1 N

c. 7.7 N

d. 8.8 N
44. A uniform rod (AB) of length L and weight W is pinned at point C. An initial impulse starts the rod accelerating
with an initial angular acceleration (in rad/s2) of g/L. What is the initial reaction at point C?

Select one:

a. w/3

b. w/4

c. 4w/7

d. 4w/7

45. What is the radius of gyration about a horizontal axis passing through the centroid?

Select one:

a. 1.7 cm

b. 0.86 cm

c. 3.7 cm

d. 2.3 cm

46. A 153 kg car is towing a 300 kg trailer. The coefiicient of friction between all tires and the road is 0.80. The car
and trailer are traveling at 100 km/h around a banked curve of radius 200 m. What is the necessary banking angle
such that tire friction will not be necessary to prevent skidding?

Select one:

a. 36o

b. 78o

c. 21o

d. 8o
47. A 47.2-kg child is standing on the outer edge of a merry-go-round that has moment of inertia 543 kg · m2 and
radius 2.40 m. The entire system is initially rotating at 0.180 rev/s. Find the angular velocity if the child moves to a
final position 1.10 m from the center of the merry-go-round.

Select one:

a. 4.123 rev/s

b. 0.132 rev/s

c. 0.244 rev/s

d. 1.324 rev/s

48. A hollow cylinder has a mass of 2 kg, a height of 1 m, an outer diameter of 1 m, and an inner diameter of 0.8 m.
What is the cylinders mass moment of inertia about an axis perpendicular to the cylinders longitudinal axis and
located at the cylinders end?

Select one:

a. 0.79 kg m2

b. 0.87 kg m2

c. 1.49 kg m2

d. 0.41 kg m2

49. Rigid link AB is 12 m long. It rotates counterclockwise about point A at 12 rev/min. A thin disk with radius 1.75
m is pinned at its center to the link at point B. The disk rotates counterclockwise at 60 rev/min with respect to
point B. What is the maximum tangetial velocity seen by any point on the disk?
Select one:

a. 45 m/s

b. 28 m/s

c. 33 m/s

d. 6 m/s

50. A car is pulling a trailer at 100 km/h. A 5 kg cat riding on the roof of the car jumps from the car to the trailer.
What is the change in the cat's momentum?

Select one:

a. -25 N s (loss)

b. 0 N s

c. 1300 N s(gain)

d. 25 N s (gain)

51. What is the magnitude o fthe couple that exactly replaces the moment that is removed?

Select one:

a. 2.5 N m

b. 0.16 N m
c. 15 N m

d. 0.08 N m

52. Refer to a particle for which the position is defined by s(t) = 2 sin tj [tin radians]. What is the magnitude of the
particles velocity at t = 4 rad?

Select one:

a. 3.30

b. 4.12

c. 2.75

d. 2.61

53. A 2 kg mass swings in a vertical plane at the end of a 2 m cord. When = 30o, the magnitude of the tangential
velo9city of the mass is 1 m/s. What is the tension in the cord at this position?

Select one:

a. 19.6 N

b. 29.4 N

c. 18.0 N

d. 24.5 N

54. What total torque is apllied to the pulley?


Select one:

a. O

b. 230 N m

c. 300 N m

d. 280 N m

55. A fisherman cuts his boats engine as it is entering a harbor. The boat comes to a dead stop with its front end
touching the dock. The fisherman's mass is 80 kg. He moves 5 m from his seat in the back to the front of the boat
in 5 s, expecting to be able to reach the dock. if the empty boat has a mass of 300 kg, how far will the fisherman
have to jump to reach the dock?

Select one:

a. 1.3 m

b. 0.0 m

c. 5.0 m

d. 1.9 m

56. A cannonball of mass 10 kg is fired from a cannon of mass 250 kg. The initial velocity of the cannonball is 1000
km/h. All of the cannon's recoil is absorbed by a spring with a spring constant of 250 N/cm. What is the maximum
recoil distance of the cannon?

Select one:

a. 0.59 m

b. 0.35 m

c. 0.92 m

d. 0.77 m
57. The cylinder shown is acted on by couple M. Wall A is frictionless (&#181s = 0), but the coefficeint of static
friction between the cylinder and wall B is &#181s = 0.3. The cylinder has a weigh of 200 N. What is the largest
value of the couple M for which the cylinder will not turn?

Select one:

a. 96 N m

b. 31 N m

c. 72 N m

d. 48 N m

58. Whatb is the polar radius of gyration?

Select one:

a. 4.2 m

b. 4.9 m

c. 3.6 m

d. 4.0 m

59. Three concurrent forces act as shown.


If the forces are in equilibrium, and F2 is 11 N, what is the magnitude of F1?

Select one:

a. 10 N

b. 8 N

c. 12 N

d. 11 N

60. If the car described in Prob.72 moves along a track that is banked 5o, what is the smallest radius it can travel
without skidding?

Select one:

a. 47 m

b. 6 m

c. 26 m

d. 18 m

61. Find the force in member BC.

Select one:

a. 50 000 N (compression)

b. 50 000 N (tension)

c. 52 700 N (compression)

d. 16 700 N (tension)
62. A projectile is fired from a cannon with an initial velocity of 1000 m/s and at an angle of 30o from the
horizontal. What distance from the cannon will the projectile strike the ground if the point of impact is 1500 m
below the point of release?

Select one:

a. 90 800 m

b. 78200 m

c. 67300 m

d. 8200 m

63. Quantity of inertia possessed by an object or the proportion between force and acceleration

Select one:

a. Mass

b. Moment of inertia

c. Velocity

d. Momentum

64. A varying force acts on a 40 kg weight as shown in the following force versus time diagram. What is the object's
velocity at t = 4 s if the object start from

Select one:

a. 0.30 m/s

b. 0.075 m/s
c. 0.15 m/s

d. 0 m/s

65. A I kg uniform rod 1 m long is suspended from the ceiling by a frictionless hinge. The rod is free to pivot. What
is the product of inertia of the about the pivot point?

Select one:

a. 0 kg m2

b. 0.045 kg m2

c. 0.13 kg m2

d. 0.33 kg m2

66. Three forces act on a hook. Determine the magnitude of the resultant of the forces. Neglect hook bending.

Select one:

a. 1250 N

b. 989 N

c. 1510 N

d. 1140 N

67. The support force exerted on an object in contact with another stable object

Select one:

a. Normal force

b. Weight

c. Tension

d. Gravity
68. Refer to a particle whose curvilinear motions is represented by the equation s = 20t + 4t2 - 3t3. What is
particles initial velocity?

Select one:

a. 25 m/s

b. 20 m/s

c. 32 m/s

d. 30 m/s

69. What is the tension in cable AB?

Select one:

a. 250 N

b. 430 N

c. 870 N

d. 500 N

70. A 100 kg block is pulled along a smooth, flat surface by an external 500 N force. If the coefficient of friction
between the block and the surface is 0.15, what acceleration is experienced by the block due to the external force?

Select one:

a. 4.33 m/s2

b. 3.23 m/s2

c. 5.00 m/s2

d. 3.80 m/s2
71. A motorist is travelling at 70 km/h when he sees a traffic light in an intersection 250 m ahead turn red. The
light's red cycle is 15 s. The motorist wanst to enter the intersection without stopping his vehicle, just as the light
turns green. What uniform deceleration of the vehicle will just put the motorist in the intersection when the light
turns greens?

Select one:

a. 0.18 m/s2

b. 1.3 m/s2

c. 0.37 m/s2

d. 25 m/s2

72. The nuts on a collar are each tightened to 18 N m torque. 17% of this torque is used to overcome screw thread
friction. The bolts have a nominal diameter of 10 mm. The threads are a simple square cut with a pitch abgle of
15o. The coefficient of friction in the threads is 0.10. What is the approximate tensile force in each bolt?

Select one:

a. 203 N

b. 1620 N

c. 405 N

d. 132 N

73. During the time a compact disc (CD) accelerates from rest to a constant rotational speed of 477 rev/min, it
rotates through an angular displacement of 0.250 rev. What is the angular acceleration of the CD

Select one:

a. 358 rad/s2

b. 794 rad/s2

c. 126 rad/s2

d. 901 rad/s2

74. A 6.0-kg block is released from rest 80m above the ground. When it has fallen 60m its kinetic energy is
approximately:
Select one:

a. 4800 J

b. 1176 J

c. 3528 J

d. 120 J

75. A particle starting from rest experienced an acceleration of 3 m/s2 for 2 s. The particle then returned to rest in
a distance of 8 m. Assuming all accelerations were uniform, what was the total time elapsed for the particles
motion?

Select one:

a. 5.33 s

b. 4.67 s

c. 2.67 s

d. 4.00 s

76. A rope passes over a fixed sheave as shown. The two rope ends are parallel. A fixed load on one end of the
rope is supported by a constant force on the other end. The coefficient of friction between the rope and the
sheave is 0.30. What is the ratio of tensile forces in the two rope ends?

Select one:

a. 1.6

b. 1.2

c. 1.1

d. 2.6

77. In an isolated system it does not change with time when there are no forces acting on the system

Select one:

a. displacement
b. force

c. momentum

d. position

78. The coeffecicient of friction between the brqake pad and drum is 0.3. Assuming that the beam supporting the
cable drum is more than adequate for the loads involved, what load,W, can be held stat5ionary?(Insert question
#12)

Select one:

a. 100 N

b. 180 N

c. 33 N

d. 90 N

79. The elevator in a 12--story building has a mass of 1000 kg. Its maximum velocity and maximum acceleartion ar
2 m/s and 1 m/s2, respectively. A paasenger with a mass of 75 kg stands on a bathroom scale in the elevator as the
elevator ascends at its maximum acceleration. what is the scale reading just as the elevator reaches its maximum
velocity?

Select one:

a. 886 N

b. 150 N

c. 75 N

d. 811 N

80. The braced frame shown is constructed with pin-connected members and supports. All applied forces are
horizontal. What is the force in the diagonal member AB?(Insert question #10)

Select one:

a. 160 N

b. 250 N

c. 0

d. 200 N
81. An automobile travels on a perfectly horizontal, unblanked circular track of radius r. The coefficient of friction
between the tires and the track is 0.3. If the car's velocity is 10 m/s, what is the smallest radius it may travel
without skidding?

Select one:

a. 10 m

b. 50 m

c. 34 m

d. 68 m

82. A 10 kg block is resting on a horizontal circular disk (e.g., turntable) at a radius of 0.5 m form the center. The
coefficient of friction between the block and disk is 0.2. the disk begins to rotate with a uniform angular
acceleration. What is the minimum angular velocity of the plate that will cause the block to slip?

Select one:

a. 4.43 rad /s

b. 1.98 rad/s

c. 3.92 rad /s

d. 1.40 rad/s

83. A rigid body is subjecyed to three cfoncurrent, coplanar forces. What is the minimum number of independent
equations that are necessary to establish the equilibrium conditions?

Select one:

a. 3

b. 2

c. 1

d. 0

84. Two meshing spur gears are arranged such that neither gear is turning and both are in equilibrium. Gear 1 has a
radius of 4 cm. Gear 1's shaft carries a torsional moment of 65 N m from an external motor. Gear 2 has a radius of
6 cm. Assuming a 100% transmission efficiency, what torque is transmitted by the shaft of gear 2?

Select one:

a. 97.5 N m

b. 65 N m
c. 107 N m

d. 101 N m

85. Determine the force in member AG for the pin-conneted truss shown.

Select one:

a. 37 500 N (tension)

b. 31 500 N (compression)

c. 25 000 N (compression)

d. 50 000 N (tension)

86. What are the -and y-coordiantes of the centroid of the area?

Select one:

a. 3.50 cm ; 5.50 cm

b. 3.93 cm ; 4.79 cm

c. 4.00 cm ; 5.00 cm

d. 3.40 cm ; 5.60 cm
87. An ideal spring is hung vertically from the ceiling. When a 2.0-kg block hangs at rest from it the spring is
extended 6.0 cm from its relaxed length. A upward external force is then applied to the block to move it upward a
distance of 16 cm. While the block is moving upward the work done by the spring is

Select one:

a. -2.09 J

b. -1.75 J

c. -1.05 J

d. -0.52 J

88. Refer to a particle for which the position is defined by s(t) = 2 sin tj [tin radians]. What is the magnitude of the
particle's acceleartion at t = π?

Select one:

a. 2.00

b. 2.56

c. 4.00

d. 3.14

89. A satellite is placed in a circular orbit to observe the surface of Mars from an altitude of 144 km. The equatorial
radius of Mars is 3397 km. If the speed of the satellite is 3480 m/s, what is the magnitude of the centripetal
acceleration of the satellite?

Select one:

a. 2.99 m/s2

b. 2.17 m/s2

c. 3.42 m/s2

d. 2.60 m/s2

90. A motorist is travelling at 70 km/h when he sees a traffic light in an intersection 250 m ahead turn red. The
light's red cycle is 15 s. The motorist wanst to enter the intersection without stopping his vehicle, just as the light
turns green. What uniform deceleration of the vehicle will just put the motorist in the intersection when the light
turns greens?

Select one:

a. 0.37 m/s2

b. 25 m/s2
c. 0.18 m/s2

d. 1.3 m/s2

91. The location of a particle moving in the -y plane is given by the parametric equations = t2 + 4t and y =(1/4)t4 -
60t, where and y are in meters and t is in seconds. What I sthe particles velocity at t = 4 s?

Select one:

a. 16.0 m/s

b. 8.95 m/s

c. 11.3 m/s

d. 12.6 m/s

92. The two cables shown carry a 100 N vertical load. What is the tension in cable AB?

Select one:

a. 80 N

b. 60 N

c. 50 N

d. 40 N

93. The pedestrian bridge truss shown has 10 000 N applied loads at points I,J, and K. What is the force in member
IJ?

Select one:
a. 18 000 N (compression)

b. 8000 N (tension)

c. 8000 N (compression)

d. 18 000 N (tension)

94. A projectile whose mass is 10 g is fired directly upward from ground level with an initial velocity of 1000 m/s.
Neglect the effects of air resistance, what will be speed of the projectile when it impacts the ground?

Select one:

a. 981 m/s

b. 1414 m/s

c. 1000 m/s

d. 707 m/s

95. The 285 kg plate shown is suspended horizontally by four wires of equal loenght, and the tension of each wire
is equal. If wire D snaps, the tension in the three remaining wires is redistributed. Determined the tension in each
wire after wire D snaps.

Select one:

a. TA /= 699 N ; TB /= 699 N ; TC /= 1398 N

b. TA /=1398 N ; TB /= 1398 N ; TC /= 0 N

c. TA /

d. TA /= 1398 N ; TB /= 0 N ; TC /= 1398 N

e. TA = 699 N ; TB = 1398 N ; TC = 699 N


96. Identfy the zero-force members in the truss shown.

Select one:

a. AB,GH,GI,HI,EG

b. AB,GH

c. AB,HI,GI

d. GI,HI

97. Three coplanar forces are in equilibrium on the surface of a steel plate, as shown. Two of the forces are known
to be 10 N. What is the angle, , of the third force?

Select one:

a. 82.5o

b. 26.7o

c. 53.8o

d. 7.50o

98. A signal arm carries two traffic signals and a sign, as shown. The siognals and sign are rigidly attached to the
arm. Each traffic signal is 0.2 m2 in frontal area and weighs 210 N. The sign weighs 60 N/m2. The design wind
pressure is 575 N/m2. The maximum moment that the connection between the arm and pole can withstand due to
wind is 6000 N m , and the maximum permitted moment due to the loads is 4000 N m. As limited by moment on
the connection, what is the maximum area of the sign?
Select one:

a. 5.65 m2

b. 1.15 m2

c. 8.03 m2

d. 1.04 m2

99. A uniform thin disk has a radius of 30 cm and a mass of 2 kg. A constant force of 10 N is applied tangentially at
a varying, but unknown, distance from the center of the disk. The disk accelerates about its axis at 3t rad/s2. What
is the distance from the center of the disk at which the force is apllied at t = 12 s?

Select one:

a. 108 cm

b. 32.4 cm

c. 54.0 cm

d. 36.0 cm

100. Four bolts (not shown) connect support A to the ground. Determine the design load for each o fthe four bolts.
Select one:

a. 300 000N (tension)

b. 50 000 N (tension)

c. 37 500 N (tension)

d. 350 000 N (tension)

101. A projectile has an initial velocity of 110 m/s and a launch angle of 20o from the horizontal. The surrounding
terrain is level, and air friction is to be disregarded. What is the horizontal distance traveled by the projectile?

Select one:

a. 1200 m

b. 80 m

c. 800 m

d. 400 m

102. QUEST032::Figure shows a uniform disk, with mass M = 2.5 kg and radius R = 20 cm, mounted on a fixed
horizontal axle. A block with mass m = 1.2 kg hangs from a massless cord that is wrapped around the rim of the
disk. Find the acceleration of the falling block. The cord does not slip, and there is no friction at the axle.

Select one:

a. -4.8 m/s2

b. 4.8 m/s2

c. -3.2 m/s2
d. 3.2 m/s2

103. Find the velocity of block A 2.5 s after the blocks are released.

Select one:

a. 3.5 m/s

b. 0 m/s

c. 4.4 m/s

d. 4.9 m/s

104. A box has uniform density and a total weight of 600 N. It is suspended by three equal-length cables, AE,BE,
and CE, as shown. Point E is 0.5 m directly above the center of the box's top surface. What is the tension in cable
CE?

Select one:

a. 400 N

b. 200 N

c. 128 N

d. 370 N

105. The five forces shown act at point A. What is the magnitude of the resultant force?

Select one:
a. 234 N

b. 182 N

c. 156 N

d. 32 N

106. Determine the reaction at point C.

Select one:

a. -417 N (down)

b. + 83 N (down)

c. +333 N (up)

d. -83 N (down)

107. A 50 kg cylinder has a height of 3 m and a radius of 50 cm. The cylinder sits on the -axis and is oriented with its
major axis parallel to the y-axis. What is the mass moment of inertia about the -axis?

Select one:

a. 41 kg mo

b. 16 kg mo

c. 150 kg mo

d. 4.1 kg mo
108. A rope is wrapped over a 6 cm diameter pipe to support a bucket of tools being lowered. The coefficient of
friction between the rope and the pipe is 0.20. The combined mass of bucket and tools is 100 kg. What is the range
of force that can be applied to the free end of the rope such that the bucket remains stationary?

Select one:

a. 720 N to 1360 N

b. 560 N to 1360 N

c. 720 N to 1510 N

d. 670 N to 1440 N

109. A model T-beam is constructed from five balsa boards. Refer to the illustration for the as-built dimensions.
What is the approximate centroidal moment of inertia about an axis parallel to the -axis?

Select one:

a. 660 cm4

b. 600 cm4

c. 500 cm4

d. 560 cm4
110. A 3 kg disk with a diameter of 0.6 m is rigidly attached at point B to 1 kg rod 1 m in length. The rod-disk
combination rotates around point A. What is the mass moment of inertia about A for the combinanation

Select one:

a. 0.56 kg m2

b. 0.87 kg m2

c. 047 kg m2

d. 3.7 kg m2

111. A 2.5-kg ball and a 5.0-kg ball have an elastic collision. Before the collision, the 2.5-kg ball was at rest and the
other ball had a speed of 3.5 m/s. What is the kinetic energy of the 2.5-kg ball after the collision?_____"

Select one:

a. 27 J

b. 14 J

c. 5.8 J

d. 8.1 J

112. A spring has a constant of 50 N/m. The spring is hung vertically, and a mass is attached to its end. The spring
end displaces 30 cm from its equilibrium position. The same mass is removed from the first spring and attached to
the end of a second (different) spring, and the displacement is 25 cm. What is the spring constant of the second
spring?

Select one:

a. 63 N/m

b. 56 N/m

c. 60 N/m

d. 46 N/m
113. What is the polar moment of nertia about the composite centroid?

Select one:

a. 1530 m4

b. 1020 m4

c. 2410 m4

d. 1260 m4

114. An area is a composite of a semicircle and a triangle, as shown. What is the distance between the -axis an
dthe centroid?

Select one:

a. 3.46 mm

b. 3.68 mm

c. 5.35 mm

d. 4.28 mm

115. Find the velocity at position B.


Select one:

a. 9.83 m/s

b. 6.95 m/s

c. 2.41 m/s

d. 4.12 m/s

116. In the structure shown, the beam is pinned at point B. Point E is a roller support. The beam is loaded with a
distributed load from point A to point B of 400 N/m, a 500 N m couple at point C, and a vertical 900 N force at
point D. If the distributed load and the vertical load are removed and replaced with a vertical upward force of 1700
N at point F, what moment at point F would be necessary to keep the reaction at point E at the same?.

Select one:

a. -6500 N m (counterclockwise)

b. 12 000 N m (clockwise)

c. 3500 N m (clockwise)

d. -9000 N m (counterclockwise)

117. What is the magnitude of the forces that constitute the moment?

Select one:

a. 8.3 N

b. 6.3 N

c. 4.2 N

d. 2.1 N
118. A 28 mm diameter circuit area is reduced by a 21 mm diameter circular area that is cut out. Both circles are
tangent to the y-axis. What is the moment of inertia about the y-axis of the remaining (shaded) area?

Select one:

a. 103 000 mm4

b. 330 000 mm4

c. 1340 000 mm4

d. 20 600 mm4

119. What is the coefficient of friction between the plane and the block?

Select one:

a. 0.15

b. 0.78

c. 0.22

d. 0.85

120. Two blocks are connected over a pulley. The mass of block A is 10 kg and the coefficient of kinetic friction
between A and the incline is 0.20. Angle θ of the incline is 30°. Block A slides down the incline at constant speed.
What is the mass of block B

Select one:

a. 2.1 kg

b. 5.0 kg

c. 3.7 kg

d. 3.3 kg
121. The center of gravity of a roller coaster car is 0.5 m above the rails. The rails are 1 m part. What is the
maximum speed that the car can travel around an unbanked curve of radius 15 m without the inner wheel losing
contact with the top of the rail?

Select one:

a. 8.58 m/s

b. 17.2 m/s

c. 24.2 m/s

d. 12.1 m/s

122. Two particles are fixed to an x axis : particle 1 of charge -2.00 x 10-7 C at x=6.00 cm and particle 2 of charge
+2.00 x 10-7 C at x=21.0 cm. Midway between the particles, what is their net electric field in unit vector notation?

Select one:

a. -3.20 X 105 N/C i ̂

b. -6.39 x 105 N/C i ̂

c. -4.00 x 105 N/C i ̂

d. -2.40 x 105 N/C i ̂

123. Find the force in member DE.

Select one:

a. 8800 N (tension)

b. 10 000 N (compresiion)

c. 0

d. 6300 N (tension)

124. A force is defined by the vector A = 3.5 i - 1.5 j + 2.0k. i,j, and k are unit vectors in the -,y-, and z-direction,
respectively. What is the angle that the force makes with the positive y-axis?
Select one:

a. 69.6o

b. 20.4o

c. 110o

d. 66.4o

125. The position (in radians) of a car travelling around a curve is described by the following function of time (in
seconds). What is the angular velocity at t = 3 s?

Select one:

a. -16 rad/s

b. -4 rad /s

c. 15 rad/s

d. 11 rad/s

126. A stone is dropped down a well. 2.47 s after the stone is realeased, a splash is heard. If the velocity of sound in
air is 342 m/s, find the distance to the surface of the water in the well.

Select one:

a. 38 m

b. 2.4 m

c. 28 m

d. 7.2 m
127. The maximum kinetic and potential energy of a spring when stretched at various displacements is equal to

Select one:

a. 1 /2Kx2

b. mg

c. –mx2

d. None of the choices

128. A car with a mass of 1530 kg tows a trailer (mass of 200 kg) at 100 km/h. What is the total momentum of the
car-trailer combination?

Select one:

a. 46 000 N s

b. 22 N s

c. 37 N s

d. 48 000 N s

129. If W = 80 N, what are the reactions

at pont A?
Select one:

a. 27 i N - 100j N

b. -27 i N - 100j N

c. 0 i N+ 180j N

d. 0 i N + 100j N

130. A parallel-plate air capacitor is made from two plates 0.070 m square, spaced 6.3 mmapart. What must the
potential difference between the plates be to produce an energydensity of 0.037 J/m3?

Select one:

a. 470 V

b. 370V

c. 270 V

d. 570 V

131. Block d side freely on the homogeneous bar and experiences a gravitation force of 50 N. Homogeneous bar
AB experiences a gravitational force of 25 N. What is the force between the bar and block D?

Select one:

a. 21 N

b. 19 N

c. 15 N

d. 28 N

132. A motorist is travelling at 70 km/h when he sees a traffic light in an intersection 250 m ahead turn red. The
light's red cycle is 15 s. The motorist wanst to enter the intersection without stopping his vehicle, just as the light
turns green. If the vehicle decelerates at a constant rate of 0.5 m/s2, what will be its speed when the light turns
green?

Select one:

a. 52 km/h

b. 63 km/h

c. 43 km/h

d. 59 km/h

133. Which of the structures shown is statically determinant and stable?

Select one:

a. I and III

b. I and IV

c. I only

d. II and III

134. The rotor of a steam turbine is rotating at 7200 rev/min when the steam supply is suddenly cut off. The rotor
decelerates at a constant rate and comes to rest after 5 min. What was the angular deceleration of the rotor?

Select one:

a. 2.5 rad/s2

b. 5.8 rad/s2

c. 0.40 rad/s2

d. 16 rad/s2
135. A 6 kg sphere moving at 3m/s collides with a 10 kg sphere traveling 2.5 m/s in the same direction. The 6 kg
ball comes to a complete stop after the collision. What is the new velocity of the 10 kg ball immediately after the
collision?

Select one:

a. 0.5 m/s

b. 5.5 m/s

c. 2.8 m/s

d. 4.3 m/s

136. Which type of load is not resisted by a pinned joint?

Select one:

a. compression

b. moment

c. shear

d. axial

137. What is the resultant R of the system of forces shown?

Select one:

a.
b.

c.

d.

138. If the frame is pinned so that it rotates around point B, what counteracting moment must be applied at point
A to put the frame in equilibrium?

Select one:

a. 1150 N m

b. 1240 N m
c. 650 N m

d. 890 N m

140. An isolated parallel-plate capacitor (not connected to a battery) has a charge of Q = 2.9 × 10-5 C. The
separation between the plates initially is d = 1.2 mm, and for this separation the capacitance is 3.1 × 10-11 F.
Calculate the work that must be done to pull the plates apart until their separation becomes 5.3 mm, if the charge
on the plates remains constant. The capacitor plates are in a vacuum

Select one:

a. 5 J

b. 46 J

c. 48 J

d. 47 J

141. A wheel with a radius of 80 cm rolls along a flat surface at 3 m/s. If arc AB on the wheels perimeter measures
90o, what is the velocity of point A when point B contacts the ground?

Select one:

a. 3.39 m/s

b. 3.75 m/s

c. 4.24 m/s

d. 3.00 m/s

142. A disk-shaped bofy with a 4 cm radius has a 320 N force acting through the center at an unknown angle , and
two 40 N loads acting as a couple, as shown. All of these forces are removed and replaced by a single 320 N force
at point B, parallel to the original 320 N force. What is the angle ?
Select one:

a. 0o

b. 7.6o

c. 15o

d. 29o

143. A block with a mass of 150 kg is pulled over a horizontal surface by a cable guided by a pulley as shown. The
coefficients of friction are 0.58 between the surface and the block, and 0.90 between the cable and the pulley.
What force,F, must be applied to the cable for the block to move?

Select one:

a. 2500 N

b. 900 N

c. 1700 N

d. 2200 N

144. Refer to a particle whose curvilinear motions is represented by the equation s = 20t + 4t2 - 3t3. What is the
acceleration of the particle at time t = 0?

Select one:

a. 2 m/s2

b. 5 m/s2

c. 3 m/s2

d. 8 m/s2

145. A playground merry-go-round has a radius of 3.0m and a rotational inertia of 600 kg m2.It is initially spinning
at 0.80 rad/s when a 20-kg child crawls from the center to the rim. When the child reaches the rim the angular
velocity of the merry-go-round is

Select one:
a. 0.80 rad/s

b. 1.04 rad/s

c. 0.73 rad/s

d. 0.62 rad/s

146. Traffic travels at 100 km/h around a banked high-way curve with a radius of 1000m. What banking angle is
necessary such that friction will not be required to resist the centrifugal force?

Select one:

a. 46o

b. 2.8o

c. 4.5o

d. 1.4o

147. Resolve the 300 N force into two components, one along line p and the other along line Q. (F, P and Q are
coplanar.)

Select one:

a. Fp = 126 N ; FQ /= 272 N

b. Fp = 226 N ; FQ = 135 N

c. Fp = 186 N ; FQ /= 232 N

d. Fp = 226 N ; FQ /=212 N

148. Determine the force in member BC.

Select one:

a. 1000 N (compression)
b. 2500 N (tension)

c. 1500 N (tension)

d. 0

149. The statement “An object with constant momentum is in a state of equilibrium” is

Select one:

a. Insufficient data

b. False

c. Partly true

d. True

150. The velocity (in m/s) of a falling ball is described by the equation v = 32 + t + 6t2. What I sthe acceleration at
time t = 2 s?

Select one:

a. 25 m/s2

b. 9.8 m/s2

c. 58 m/s2

d. 32 m/s2

151. A particle has a tangential acceleration of at (represented by the equation given) when it moves around a
point in a curve with instantaneous radius of 1 m. What is the instantaneous angular velocity ( in rad/s) of the
particle?

Select one:

a. t2+cost+ 3 In |csct|

b. t2-cost+ 3 In |csct|

c. t2-cost+ 3 In |sint|

d. t2+cost+ 3 In |sint|
152. A golfer on level ground attempts to drive a gof ball across a 50 m wide pond, hitting the ball so that it travels
initially at 25 m/s. The ball travels at an initial angle of 45o to the horizontal plane. How far will the golf ball travel,
and does it clear the pond?

Select one:

a. 58 m; the ball clears the pond

b. 32 n; the ball does not clear the pond

c. 45 m; the ball does not clear the pond

d. 64 m ; the ball clears the pond

153. A 2000 kg car pulls a 500 kg trailer. The car and trailer accelerates from 50 km/h to 75 km/h at rate of 1 m/s2.
What linear impules does the car impart on the trailer?

Select one:

a. 12 500 N s

b. 3470 N s

c. 17400 N s

d. 8680 N s

154. What are R1 and R2? (insert question #11)

Select one:

a. 1250 N

b. 4000 N

c. R1 /

d. 1000 N; R2 /

e. R1 /

f. R1 /

g. 3750 N; R2 /

h. 1250 N

155. QUEST029::Two blocks are connected over a pulley. The mass of block A is 10 kg and the coefficient of kinetic
friction between A and the incline is 0.20. Angle θ of the incline is 30°. Block A slides down the incline at constant
speed. What is the mass of block B

Select one:

a. 3.7 kg

b. 3.3 kg

c. 2.1 kg

d. 5.0 kg

156. A mass of 10 kg is suspended from a vertical spring with a spring constant of 10 N/m. What is the period of
vibration?

Select one:

a. 6.3 s

b. 0.30 s

c. 0.60 s

d. 0.90 s

157. What is the reaction at point A for the simply supported beam shown?

Select one:

a.

b. None of the choices

c.
d.

158. A bent beam is acted upon by a moment and several concentrated forces, as shown. Find the missing force F
and distance that will maintain equilibrium on the member shown.

Select one:

a. F = 20 N ; = 0.2 m

b. F = 10 N ; = 0.6 m

c. F = 20 N ; = 0.4 m

d. F = 5 N ; = 0.8 m

159. A car travels around an unbanked 50 m radius curve without skidding. The coefficient of friction between the
tires and road is 0.3. What is the car's maximum speed?

Select one:

a. 54 km/h

b. 25 km/h

c. 44 km/h

d. 14 km/h

160. A force that is directed away or towards the origin

Select one:

a. Frictional force
b. Uniform force

c. Central force

d. Normal force

161. QUEST030::The small piston of a hydraulic lift has a cross-sectional area of 3.00 cm2, and its large piston has a
cross-sectional area of 200 cm2 . What force must be applied to the small piston for it to raise a load of 15.0 kN?
(In service stations, this force is usually generated with the use of compressed air.)

Select one:

a. 1.00 X 102 N

b. 40 N

c. 1.00 x 103 N

d. 225 N

162. A disk rolls along a flat surface at a constant speed of 10 m/s. Its diameter is 0.5 m. At a particular instant,
point P on the edge of the disk is 45o from the horizontal. What is the velocity of point P at that instant?
Select one:
a. 10.0 m/s15.0 m/s16.2 m/s
b. 18.5 m/s

163. For the reciprocating pump shown, the radius of the crank is r = 0.3 m, and the rotational speed is n = 350
rpm. What is the tangetial velocity of point A on the crank corresponding to an angle of = 35o from the horizontal?

Select one:

a. 10 m/s

b. 1.1 m/s

c. 0 m/s

d. 11 m/s

164. What is the -coordinate of the centroid of the curve y = cos between = 0 = /2?
a. pi/4
b. pi/6
c. 1 – 2/pi
d. pi/2 – 1

165. QUEST031::Two particles are fixed to an x axis : particle 1 of charge -2.00 x 10-7 C at x=6.00 cm and particle 2
of charge +2.00 x 10-7 C at x=21.0 cm. Midway between the particles, what is their net electric field in unit vector
notation?

Select one:

a. -6.39 x 105 N/C i ̂


b. -2.40 x 105 N/C i ̂

c. -4.00 x 105 N/C i ̂

d. -3.20 X 105 N/C i ̂

166. Refer to a particle whose curvilinear motions is represented by the equation s = 20t + 4t2 - 3t3. What is the
maximum speed reached by the particle?

Select one:

a. 34.6 m/s

b. 27.9 m/s

c. 48.0 m/s

d. 21.8 m/s

167. A torsional pendulum consists of a 5 kg uniform disk with a diameter of 50 cm attached at its center to a rod
1.5 m in length. The torsional spring constant is 0.625 N.m/rad. Disregarding the mass of the rod, what is the
natural frequency of the torsional pendulum?

Select one:

a. 1.0 rad/s

b. 1.4 rad/s

c. 1.2 rad/s

d. 2.0 rad/s

168. The position (in radians) of a car travelling around a curve is described by the following function of time (in
seconds). What is the angular acceleration at t = 5 s?

Select one:

a. 4 rad/s2

b. 26 rad/s2

c. 30 rad/s2

d. 6 rad/s2
«May the odds be ever in your favor.»

Reviewer in Chemistry

1. Rank the following gas according to increasing effusion rates relative to O2 (reference). MM F2 = 38, MM CH4 =
16, MM CO2 = 44, MM O2 = 32

2. What is the vapor pressure of 1000.0 g of a water solution at 250C that contains 124.0g of the nonvolatile
solute ethylene glycol, C2H6O2? The vapor pressure of pure water at this temperature is 23.76 torr. Assume an
ideal solution.
3. The decay of U-238 to Pb-206 can be used to estimate the age of inorganic matter. The half-life of U-238 is 4.5 x
109 years. In a particular rock sample, the ratio of the numbers of Pb-206 to U- 238 atoms is 0.66. Assume all of
the Pb-206 present is due to the decay of U-238. What is the age of the rock?

4. The diameter of a spherical mothball is observed to halve in 200 days. Approximately, how long will it take for its
remaining volume to become half of its volume at 200 days.

5. A 350mL sample of 0.276 M HNO3 is partially neutralized by 125mL of 0.0120 M Ca(OH)2. Find the
concentration of nitric acid in the resulting solution.

6. A commonly occurring isotope of tin-118, while most oxygen occurs in nature as oxygen-16. A molecule of
tin(IV) oxide formed from these isotopes contains how many neutrons?
7. A method of removing CO2 from a spacecraft is to allow the CO2 to react with sodium hydroxide. (The products
of the reaction are sodium carbonate and water). What volume of carbon dioxide at 25oC and 749 mmHg can be
removed per kilogram of sodium hydroxide that reacts?
8. The solubility of constant of stronyium sulfate, SrSO4 is 2.8 x 10 -7. How many grams of SrSO4 must be dissolved
in water to produce 1L saturated solution.

9. The mineral manganosite is a compound of manganese-55 and oxygen-16. If 77% of the mass manganosite is
due to manganese, what is the empirical formula of manganosite?

10. The first step in the Otswald process for producing nitric acid is 4NH3(g) + 5O2(g) ? 4NO(g) + 6H2O(g). If the
reaction of 150 g of ammonia with 150 g of oxygen gas yields 87 g of nitric oxide (NO), what is the percent yield of
this reaction?
Reviewer in Physics

1. A 2.5-kg ball and a 5.0-kg ball have an elastic collision. Before the collision, the 2.5-kg ball was at rest and the
other ball had a speed of 3.5 m/s. What is the kinetic energy of the 2.5-kg ball after the collision?

2. A parallel-plate air capacitor is made from two plates 0.070 m square, spaced 6.3 mm apart. What must the
potential difference between the plates be to produce an energy density of 0.037 J/m3?

3. A single force (not shown) is applied at point B in the y-direction, in line with points A and B. What should this
force being order for the frame to be in equilibrium in that direction?
4. Determine the force in member AG for the pin-connected truss shown.

5. The two cables shown carry a 100 N vertical load. What is the tension in Cable AB?
6. What are the x- and y- coordinates of the centroid of the perimeter line?
7. A particle starting from rest experienced an acceleration of 3 m/s2 for 2 s. The particle then returned to rest in a
distance of 8 m. Assuming all accelerations were uniform, what was the total time elapsed for the particles
motion?
8. The location of a particle moving in the –y plane is given by the parametric equations x= t2 + 4t and y=(1/4)t4-
60t where x and y are in meters and t is in seconds. What is the particles velocity at t=4s?

9. Two blocks are connected over a pulley. The mass of block A is 10 kg and the coefficient of kinetic friction
between A and the incline is 0.20. of the incline is 30o. Block A slides down the incline at constant speed. What is
the mass of Block B?
10. A motorist is travelling at 70km/h when he sees a traffic light in an intersection 250 m ahead turn red. The
light’s red cycle is 15 s. The motorist wants to enter the intersection without stopping his vehicle, just as the light
turns green. What uniform deceleration of the vehicle will just put the motorist in the intersection when the light
turns greens?

Algebra

1. The polynomial x3 + 4x2 – 3x + 8 is divided by x-5, then the remainder is.

2. In certain Board Examination, 119 examinees too the Shop Machinery subjected, 104 examinees took thye
Power Plant Machinery subject and 115 examinees took the Industrial Plant Machinery subject. Seventy-eight (78)
conditioned examinees took only Shop Machinery and Power Machinery subjects. Seventy-one (71) conditioned
examinees took only the Power Plant Machinery and Industrial Plant Machinery subjects. Eighty-five (85)
conditioned examinees took only Industrial Plant Machinery and Shop Machinery subjects. Fifty-four took all the
three subjects. How many examinees took the Certified Plant Mechanic board examination?
3. A certain manufactured part can be defective because it has one or more out of the three possible defects:
insufficient tensile strength, a burr, or a diameter outside of tolerance limit. In a lot of 500 pieces: 19 have a tensile
strength defects, 17 have a burr, 11 have an unacceptable diameter, 12 have tensile strength and burr defects, 7
have tensile strength and diameter defects, 5 have burr and diameter defects and 2 have all three defects.
Determine: How many of the pieces have no defects? How many pieces have only burr defects? How many pieces
have exactly 2 defects?

4. A certain company manufactures two products, X and Y, and each of these products must be processed on two
different machines. Product X requires 1 minute of work time per unit on machine 1 and 4 minutes of work time on
machine 2. Product Y requires two minutes of work time per unit on machine 1 and 3 minutes of work time per
unit on machine 2. Each day, 100 minutes are available on machine 1 and 200 minutes are available on machine 2.
To satisfy certain customers, the company must produce at least 6 units per day of product X and at least 12 units
of product Y. If the profit of each unit of product X is P50 and the profit of each unit of product Y is P60, how many
of each product should be produced in order to maximize the company's profit?
Trigonometry

1. Triangle ABC is a right triangle with the right angle at C. CD is perpendicular to AB. BC = 4, and CD = 1. Find the
area of the triangle ABC.

2. A ladder 5 m long leans against the wall of an apartment house forming an angle of 50 degrees, 32 minutes with
the ground. How high of the wall does it reach?

3. If 3x = 9y and 27y = 81 z, find x/z.


4. A corner lot of land is 35 m on one street and 25 m on the other street, the angle between the two lines of the
street being 85o24’. The other of the lot are respectively perpendicular to the lines of the streets. What is the
worth of the lot at P 180 per sq m?
Solid Mensuration and Analytic Geometry

1. Two triangles have equal bases. The altitude of one triangle is 3 units more than its base while the altitude of the
other is 3 units less than its base. Find the altitudes if the areas of the triangles differ by 21 units 2.

2. One of the diagonals of a rhombus is 25 units and its area is 75 units 2. Determine the length of the side.

3. It is desired that the volume of the sphere be tripled. By how many times will the radius be increased?

4. The area of a circle is 89.42 in2. What is the length of the side of a regular hexagon inscribed in a circle?
5. Two vertices of a triangle are (2, 4) and (-2, 3) and the area is 2 square units, the locus of the third vertex
is______.

6. Determine B such that 3x + 2y -7 = 0 is perpendicular to 2x - By + 2= 0.

7. The center of a circle is at (1, 1) and one point on its circumference is (-1, -3). Find the other end of the diameter
through ( -1, -3).
8. The major axis of the elliptical path in which the earth moves around the sun is approximately 186,000,000
miles and the eccentricity of the ellipse is 1/60. Determine the apogee of the earth.

Calculus

1. Water is running out in a conical funnel at the rate of 1 cu. In. per second. If the radius of the base of the funnel
is 4 inches and the altitude in 8 inches, find the rate at which the water level is dropping when it is 2 inches from
top.
2. What is the area of the largest rectangle that can be inscribed in a semi-circle of radius 10?

3. Three sides of a trapezoid are each 8 cm. long. How long is the fourth side when the area of the trapezoid has
the greatest value?

4. The volume of the sphere is increasing at the rate of 6 cm3 / hr. At what rate is its surface area increasing (in
cn2/hr) when the radius is 50cm?
Differential Equation and Advanced Mathematics

1. A tank initialy contains 400 liters of water. Salt solution, containing 1/8 kg of salt per liter of solution flows into
the tank at the rate of 8 li/min and the solution, kept well-stirred, flows out of the tank at the rate of 4 li/min. Find
the amount of salt in the tank after 100 minutes.

2. A body whose temperature is 180degrees is immersed in a liquid which is kept at a constant temperature of 60
degrees. In 10 minutes the temperature of the immersed body decreased to 120 degrees. How long will it take for
the body's temperature to decrease to 90o?
3. A 10-ohm resistor and a 5-henry inductor are connected in series with a 50-volt source at time t = 0. Express the
current I as a function of time.

4. What is the general solution of (D3 -3D2 - 4D + 12)y = 0

5. Solve the differential equation: x(y-1)dx + (x+1)dy = 0. If y = 2 when x = 1, determine y when x = 2.


6. Find the principal 5th root of 50(cos 150° + j sin 150°). 7. The indicial equation of the differential equation x 2
y"+xy'+(x 2 -4)y = 0 is 8. The Fourier cosine transform of f(x) = e -2x

7. The indicial equation of the differential equation x 2 y"+xy'+(x 2 -4)y = 0 is

8. The Fourier cosine transform of f(x) = e -2x

You might also like